Как составит уравление

Решение задач составлением уравнения

Разделы: Математика

«Умственную самодеятельность, сообразительность и смекалку нельзя ни «вдолбить», ни «вложить» ни в чью голову. Результаты надёжны лишь тогда, когда введение в область математических знаний осуществляется в лёгкой и приятной форме, на предметах и примерах обыденной и повседневной обстановки, подобранных с надлежащим остроумием и занимательностью».

Е.И. Игнатьев «В царстве смекалки»

Цель обучающая: Научить анализировать условие задачи, выбирать рациональные способы решения, составлять уравнение, решать его, проверять правильность решения.

Цель воспитательная: Развитие абстрактного и логического мышления.

I. Актуализация опорных знаний.

1) Что называется уравнением? Что называется корнем уравнения? Что значит решить уравнение?

2) Решите уравнение .

Решение: а) ; ; x= ;

б) НОЗ=6; ; ; ; ;

3) Составьте буквенное выражение.

Наташа купила блокнот за m рублей, книгу на 20 рублей дороже блокнота и ручку в 2.5 раза дешевле книги. Сколько рублей стоит ручка?

Решение: (m+20) рублей цена книги, рублей цена ручки.

4) Из двух сёл, расстояние между которыми 10 км навстречу друг другу вышли мальчик и девочка и встретились через 2 часа. Скорость мальчика 3 км/час. Найдите скорость девочки.

Решение: а) 3*2=6(км) прошел мальчик, 10-6=4(км) прошла девочка, 4:2=2(км/ч) скорость девочки

б) 10:2=5(км/ч) скорость сближения, 5-3=2(км/ч) скорость девочки.

II. Сообщение темы и цели урока.

III. Решение задач составлением уравнения.

1) Андрей старше Олега на 4 года, а Олег старше Бориса в 1,5 раза. Вместе им 36 лет. Сколько лет каждому из них?

Первый ряд решает задачу, взяв за неизвестную величину возраст Андрея, второй ряд — возраст Олега, третий ряд-возраст Бориса. А затем каждый ряд объясняет своё решение у доски.

Условие задачи Решение уравнения Проверка
Х лет Андрею,

лет Борису,

16+12+8=36
Х лет Олегу

лет Борису

12+16+8=36
Х лет Борису

(1,5х+4) лет Андрею

8+12+16=36

Если в задаче несколько неизвестных величин, лучше обозначить буквой наименьшую из них.

2) Два пешехода вышли одновременно навстречу друг другу из двух посёлков и встретились через 3ч. Расстояние между посёлками 30 км. Найдите скорость каждого пешехода, если у одного она на 2 км/ч меньше, чем у другого.

а) Ученики решают задачу составлением уравнения.

Х км/ч – скорость 1-го пешехода

(х+2) км/ч – скорость 2-го пешехода

3х км прошел 1-й пешеход

3(х+2) км прошел 2-й пешеход

3х+3(х+2)=30 4
3х+3х+6=30 +6
6х+6=30 +12
6х=24 +18
Х=4 =30

б) Затем решают арифметическим способом.

10-2=8(км/ч)-две скорости 1-го пешехода.

8:2=4(км/ч)-скорость 1-го пешехода.

4+2=6(км/ч)-скорость 2-го пешехода.

Вывод. Арифметическое решение задачи более рациональное

3)Для распечатки 340 страниц были использованы две копировальные машины. Первая машина работала 10 минут, а вторая 15 минут. Сколько страниц в минуту печатает каждая машина, если первая печатает на 4 страницы больше, чем вторая?

Снова дети решают задачу алгебраически и арифметически.

А) х страниц за 1 минуту печатает 2-я машина,

(х+4) страниц за 1 минуту напечатает 1-я машина,

15х страниц напечатает 2 машина,

10(х+4) страниц напечатает 2 машина,

15х+10(х+4=340 12
15х+10х+40=340 +16
25х+40=340 +180
25х=300 +160
Х=12 =340

б) 10*4=40(стр) на столько страниц больше напечатает 1-я машина за 10 минут

340-40=300(стр) напечатали бы обе машины вместе, если бы у них были одинаковые скорости.

10+15=25(мин) работали обе машины.

300:25=12(стр) за 1-у минуту печатает 2-я машина.

12+4=16 (стр) за 1-у минуту печатает 1-я машина.

При решении задач с помощью уравнения поступают следующим образом:

  1. Внимательно читают условие задачи.
  2. Обозначают неизвестную величину буквой.
  3. Переводят условие задачи на алгебраический язык.
  4. Составляют уравнение.
  5. Решают уравнение.
  6. Проверяют правильность решения

V. Домашнее задание.

Решить задачу алгебраически и арифметически.

Охотничья собака спугнула зайца, который сидел под кустом в 150м от неё. Через сколько минут собака догонит зайца, если она пробегает за 6 минут 3,6км, а заяц только 3км?

  1. Л.В. Кузнецова, Е.А. Бунимович, Б.П. Пигарев, С.Б. Суворова «Алгебра» сборник заданий для проведения письменного экзамена по алгебре за курс основной школы. Москва. «Дрофа». 2001г; стр 43, работа №31, вариант 1(5); стр 50, работа №38, вариант 2(7); стр 65, работа №48, вариант 1(6).
  2. Составители Т.А. Братусь, Н.А. Жарковская, Е.А Рисс, Т.Е Савелова «Детский математический календарь 2001-2002». Санкт — Питербург стр10.
  3. Н.Т. Кострикина «Задачи повышенной трудности в курсе алгебры 7-9 классов». Москва. «Просвещение» 1991 стр5-19.

Решение задач с помощью уравнений

Тема урока: § 6. Решение задач с помощью уравнений. Приведены все необходимые и достаточные сведения для решения текстовых задач с помощью составления уравнений.

Введение

В школьной математике есть целый кладезь текстовых задач, которые решаются универсальным методом построения уравнения (модели) исходя из условия.

Сам факт того, что огромное количество самых разнообразных задач поддаются решению с помощью составления линейного уравнения, говорит нам, что метод решений является действительно универсальным.

Обычно условия задач удается перевести на математический язык. Полученное уравнение — это следствие перевода нашего условия с русского языка на язык алгебры. Зачастую фактической стороной повествования задачи является описание реальной ситуации, какого либо процесса, события.

Чтобы получить ответ — уравнение нужно решить, полученный корень уравнения будет являться решением, разумеется необходимо еще проверить, не является ли результат противоречивым относительно условия.

Алгоритм решения текстовых задач с помощью уравнений

Для решения задачи с помощью уравнения делают следующие действия:

  1. Обозначают некоторое неизвестное буквой и, пользуясь условием, составляют уравнение.
  2. Решают уравнение.
  3. Истолковывают результат.

Примеры решений

Задача 1.
В мешке было в 3 раза меньше монет, чем в сундуке. После того как из мешка переложили 24 монеты, в сундуке их стало в 7 раз больше, чем в мешке. Сколько было монет в мешке и сколько в сундуке?

Пусть $x$ — количество монет в мешке, а значит в сундуке: $3x$ монет. После того, как из мешка переложили $24$ монеты, в сундуке стало: $3x+24$, а в мешке $x-24$. И если в сундуке их стало в $7$ раз больше чем в мешке, то имеем: $3x+24=7(x-24)$.

Ну вот мы и составили уравнение (математическую модель), осталось решить уравнение относительно $x$ и записать ответ.

Решим полученное уравнение: $3x+24=7(x-24)$. Легко увидеть, что уравнение является линейным (узнать как решаются линейные уравнения можно тут.)

Раскроем скобки в правой части уравнения: $3x+24=7x-7cdot 24$. Перенесём все слагаемые содержащие переменную в правую часть, а всё что не содержит $x$ в левую, получим: $24+7cdot 24=7x-3x$. После упрощения получили $192=4x$, разделим обе части уравнения на коэффициент при неизвестном, т.е на $4$, тогда получим $x=48$.

Осталось истолковать ответ.
За переменную $x$ мы обозначали количество монет в мешке, значит в сундуке в три раза больше т.е $3x$.

Монет в мешке: $48$

Монет в сундуке: $48cdot 3=144$

Задача 2.
Купили 3600 кг муки и высыпали её в три мешка. В первый мешок муки вошло в 3 раза больше, чем во второй, а в третий мешок насыпали 800 кг муки. Сколько муки насыпали в первый и сколько во второй мешок?

Пусть в первый мешок насыпали $3x$ кг муки, тогда во второй мешок насыпали $x$ кг. Если сложим количество кг в каждом мешке, то получим $3600$ кг муки. Имеем: $3x+x+800=3600$, решим уравнение классическим методом.

Все слагаемые содержащие $x$ оставим слева, а всё остальное перенесём в правую часть равенства: $3x+x=3600-800$, упростим обе части; $4x=2800$ поделим обе части равенства на $4$ и получим ответ: $x=700$.

Ответ.
За переменную $x$ мы обозначали количество муки во втором мешке, по условию в первом в три раза больше.

Муки в первом мешке: $700cdot 3=2100$ кг.

Муки во втором мешке: $700$ кг.

Задача 3.
В первом мешке в 4 раза больше картофеля, чем во втором. После того, как из одного мешка взяли 40 кг картофеля, а во второй насыпали ещё 5 кг, в обоих мешках картофеля стало поровну. Сколько килограммов картофеля было во втором мешке.

Пусть во втором мешке $x$ кг картофеля, тогда в первом мешке $4x$ кг. Из первого взяли $40$ кг, тогда в первом стало: $4x-40$. Во второй мешок насыпали $5$ кг и теперь в нём: $x+5$ кг картошки. Нам известно, что после этих изменений количество картофеля в мешках стало поровну, запишем это с помощью линейного уравнения:

Решим это линейное уравнение. Все слагаемые содержащие переменную перенесём влево, а свободные члены вправо и получим:

Избавимся от коэффициента при неизвестном и получим ответ:

Ответ.
За переменную $x$ мы обозначали количество кг картошки во втором мешке, по условию в первом в четыре раза больше.

Картошки в первом мешке: $15cdot 4=60$ кг.

Картошки во втором мешке: $15$ кг.

Задача 4.
По шоссе едут две машины с одной и той же скоростью. Если первая увеличит скорость на 20 км/ч, а вторая уменьшит скорость на 20 км/ч, то первая за 2 часа пройдёт то же самое расстояние, что и вторая за 4 часа. Найдите первоначальную скорость машин.

Пусть машины едут со скоростью $v$ км/ч, тогда после ускорения первой машины её скорость стала: $v+20$ км/ч, а скорость второй машины после замедления стала: $v-20$ км/ч. Нам известно по условию, что после изменения скоростей машин, первая проходит за два часа ровно столько, сколько вторая за четыре, тогда имеем:

По известной нам формуле $S=vt$ ($S$ — расстояние, $v$ — скорость, $t$ — время)

Сократим обе части равенства на $2$, тогда получим: $v+20=2(v-20)$. Раскроем скобки в правой части уравнения и сгруппируем все переменные в правой части равенства.

Ответ.
В качестве неизвестной величины в задаче мы взяли $v$ (первоначальную скорость машин).

Первоначальная скорость машин: $v=60$ км/ч.

Задача 5.
В первую бригаду привезли раствора цемента на 50 кг меньше, чем во вторую. Каждый час работы первая бригада расходовала 150 кг раствора, а вторая – 200кг. Через 3 ч работы в первой бригаде осталось раствора в 1,5 раза больше, чем во второй. Сколько раствора привезли в каждую бригаду?

Пусть во вторую бригаду привезли $x$ кг раствора цемента, тогда в первую бригаду привезли $x-50$ кг. Через 3 часа работы у первой бригады осталось $x-50-3cdot 150$ кг цемента, а у второй $x-3cdot 200$ кг.

По условию известно, что через 3 часа работы в первой бригаде осталось в 1,5 раза больше цемента, чем во второй, тогда имеем:

$$x-50-3cdot 150=1,5(x-3cdot 200)$$

Осталось решить данное уравнение относительно $x$ и истолковать ответ.

Упростим и раскроем скобки в правой части, тогда получим:

Если вам неудобно работать с десятичными дробями, то вы всегда можете их переводить в рациональный вид: $1,5=frac<15><10>=frac<3><2>$.

Запишем с учётом перевода дробей и упростим:

Перенесём слагаемые содержащие переменную в правую сторону, а всё остальное в левую:

Домножим обе части на 2 и получим ответ:

Ответ.
В качестве переменной в задаче мы взяли $x$ (кол-во кг цемента который привезли во вторую бригаду), по условию в первую привезли на 50 кг меньше, а значит $x-50$

Кол-во цемента в первой бригаде: $800-50=750$ кг.

Кол-во цемента во второй бригаде: $800$ кг.

Задачи для самостоятельного решения

По контракту работникам причитается 48 франков за каждый отработанный день, а за каждый неотработанный день с них вычитается по 12 франков. Через 30 дней выяснилось, что работникам ничего не причитается. Сколько дней они отработали в течение этих 30 дней?

Пусть работники отработали $n$ дней, тогда $30-n$ дней они не отработали.

В итоге мы понимаем, что за $n$ рабочих дней они зарабатывают $48n$ франков и с них вычитается за $30-n$ не отработанных дней по $12(30-n)$ франков. Тогда ясно, что: $48n-12(30-n)=0$

Ответ: Рабочие отработали 6 дней.

Кирпич весит фунт и полкирпича. Сколько фунтов весит кирпич?

Пусть целый кирпич весит весит $k$ фунтов, тогда имеем:

1 фунт и половина кирпича = целый кирпич.

Бутылка с пробкой стоит 10 копеек, причем бутылка на 9 копеек дороже пробки. Сколько стоит бутылка без пробки?

Пусть бутылка стоит $b$ копеек, а пробка $p$ копеек, тогда:

$b+p=10$ и $b=p+9$, подставив значение $b$ в первое равенство — получим:

Т.е пробка стоит пол копейки, тогда бутылка $9,5$ копеек.

Ответ: 9,5 копеек стоит бутыка без пробки.

На свитер, шапку и шарф израсходовали 555 г шерсти, причем на шапку ушло в 5 раз меньше шерсти, чем на свитер, и на 5 г больше, чем на шарф. Сколько шерсти израсходовали на каждое изделие?

Пусть на свитер потратили $5x$ г шерсти, тогда на шапку ушло $x$ г и на шарф потребовалось $x-5$ г, имеем:

Ответ: На шапку ушло $80$ г, на свитер $5cdot 80=400$ г, на шарф $80-5=75$ г.

Три пионерских звена собрали для школьной библиотеки 65 книг. Первое звено собрало на 10 книг меньше, чем второе, а третье — 30% того числа книг, которое собрали первое и второе звено вместе. Сколько книг собрало каждое звено?

Пусть второе звено собрало $x$ книг, тогда первое собрало $x-10$ книг, а третье $0,3(2x-10)$, имеем:

$$2x-10+0,3cdot 2x-0,3cdot 10=65$$

$$2x+0,3cdot 2x=65+10+0,3cdot 10$$

Ответ: Первое звено собрало $30-10=20$ книг, второе $30$ книг, третье $0,3(60-10)=15$ книг.

Решение задач с помощью уравнений

Решение задачи обычно свóдится к тому, чтобы путем логических рассуждений и вычислений найти значение какой-нибудь величины. Например, найти скорость, время, расстояние, массу какого-нибудь предмета или количество чего-то.

Такую задачу можно решить с помощью уравнения. Для этого искомое значение обозначают через переменную, затем путем логических рассуждений составляют и решают уравнение. Решив уравнение, производят проверку на то, удовлетворяет ли решение уравнения условиям задачи.

Запись выражений, содержащих неизвестное

Решение задачи сопровождается составлением уравнения к этой задаче. На начальном этапе изучения задач желательно научиться составлять буквенные выражения, описывающие ту или иную жизненную ситуацию. Этот этап не является сложным и его можно изучать в процессе решения самой задачи.

Рассмотрим несколько ситуаций, которые можно записать с помощью математического выражения.

Задача 1. Возраст отца x лет. Мама на два года младше. Сын младше отца в 3 раза. Запишите возраст каждого с помощью выражений.

Решение:

Задача 2. Возраст отца x лет, мама на 2 года младше отца. Сын младше отца в 3 раза, дочь младше матери в 3 раза. Запишите возраст каждого с помощью выражений.

Решение:

Задача 3. Возраст отца x лет, мама на 3 года младше отца. Сын младше отца в 3 раза, дочь младше матери в 3 раза. Сколько лет каждому, если общий возраст отца, мамы, сына и дочери составляет 92 года?

Решение:

В данной задаче помимо записи выражений, необходимо вычислить возраст каждого члена семьи.

Сначала запишем возраст каждого члена семьи с помощью выражений. За переменную x примем возраст отца, и далее пользуясь этой переменной составим остальные выражения:

Теперь определим возраст каждого члена семьи. Для этого нам нужно составить и решить уравнение. Все компоненты уравнения у нас уже готовы. Осталось только собрать их воедино.

Общий возраст в 92 года получился путем сложения возрастов папы, мамы, сына и дочери:

Для каждого возраста мы составили математическое выражение. Эти выражения и будут компонентами нашего уравнения. Давайте соберем наше уравнение согласно данной схеме и таблице, которая была приведена выше. То есть слова папа, мама, сын, дочь заменим на соответствующее им в таблице выражение:

Выражение, отвечающее за возраст мамы x − 3, для наглядности было взято в скобки.

Теперь решим получившееся уравнение. Для начала можно раскрыть скобки там, где это можно:

Чтобы освободить уравнение от дробей, умножим обе части на 3

Решим получившееся уравнение, пользуясь известными тождественными преобразованиями:

Мы нашли значение переменной x . Эта переменная отвечала за возраст отца. Значит возраст отца составляет 36 лет.

Зная возраст отца, можно вычислить возрасты остальных членов семьи. Для этого нужно подставить значение переменной x в те выражения, которые отвечают за возраст конкретного члена семьи.

В задаче было сказано, что мама на 3 года младше отца. Ее возраст мы обозначили через выражение x−3. Значение переменной x теперь известно, и чтобы вычислить возраст мамы, нужно в выражении x − 3 вместо x подставить найденное значение 36

x − 3 = 36 − 3 = 33 года маме.

Аналогично определяется возраст остальных членов семьи:

Проверка:

Задача 4. Килограмм яблок стоит x рублей. Запишите выражение, вычисляющее сколько килограмм яблок можно купить на 300 рублей.

Решение

Если килограмм яблок стоит x рублей, то на 300 рублей можно купить килограмм яблок.

Пример. Килограмм яблок стоит 50 рублей. Тогда на 300 рублей можно купить , то есть 6 килограмм яблок.

Задача 5. На x рублей было куплено 5 кг яблок. Запишите выражение, вычисляющее сколько рублей стоит один килограмм яблок.

Решение

Если за 5 кг яблок было уплачено x рублей, то один килограмм будет стоит рублей

Пример. За 300 рублей было куплено 5 кг яблок. Тогда один килограмм яблок будет стоит , то есть 60 рублей.

Задача 6. Том, Джон и Лео на перемене пошли в столовую и купили по бутерброду и по кружке кофе. Бутерброд стоит x рублей, а кружка кофе — 15 рублей. Определите стоимость бутерброда, если известно, что за всё было уплачено 120 рублей?

Решение

Конечно, данная задача проста как три копейки и ее можно решить не прибегая к уравнению. Для этого из 120 рублей нужно вычесть стоимость трех кружек кофе (15 × 3) , и полученный результат разделить на 3

Но наша цель — составить уравнение к задаче и решить это уравнение. Итак, стоимость бутерброда x рублей. Куплено их всего три. Значит увеличив стоимость в три раза, мы получим выражение описывающее сколько рублей было уплачено за три бутерброда

3x — стоимость трех бутербродов

А стоимость трех кружек кофе можно записать как 15 × 3 . 15 это стоимость одной кружки кофе, а 3 множитель (Том, Джон и Лео), увеличивающий эту стоимость в три раза.

По условию задачи за все уплачено 120 рублей. У нас уже появляется примерная схема, что нужно делать:

Выражения, описывающие стоимость трех бутербродов и трех кружек кофе, у нас уже готовы. Это выражения 3x и 15 × 3 . Пользуясь схемой составим уравнение и решим его:

Итак, стоимость одного бутерброда составляет 25 рублей.

Задача решается верно только в том случае, если уравнение к ней составлено правильно. В отличие от обычных уравнений, по которым мы учимся находить корни, уравнения для решения задач имеют своё конкретное применение. Каждый компонент такого уравнения может быть описан в словесной форме. Составляя уравнение, обязательно нужно понимать для чего мы включаем в его состав тот или иной компонент и зачем он нужен.

Также необходимо помнить, что уравнение это равенство, после решения которого левая часть должна будет равняться правой части. Составленное уравнение не должно противоречить этой идее.

Представим, что уравнение это весы с двумя чашами и экраном, показывающим состояние весов.

В данный момент экран показывает знак равенства. Понятно почему левая чаша равна правой чаше — на чашах ничего нет. Состояние весов и отсутствие на чашах чего-либо запишем с помощью следующего равенства:

Положим на левую чашу весов арбуз:

Левая чаша перевесила правую чашу и экран забил тревогу, показав знак не равно ( ≠ ). Этот знак говорит о том, что левая чаша не равна правой чаше.

Теперь попробуем решить задачу. Пусть требуется узнать сколько весит арбуз, который лежит на левой чаше. Но как это узнать? Ведь наши весы предназначены только для проверки равна ли левая чаша правой.

На помощь приходят уравнения. Вспомним, что уравнение по определению есть равенство, содержащее в себе переменную значение которой требуется найти. Весы в данном случае играют роль этого самого уравнения, а масса арбуза это переменная, значение которой нужно найти. Наша цель правильно составить это уравнение. Понимай, выровнять весы так, чтобы можно было вычислить массу арбуза.

Чтобы выровнять весы, на правую чашу можно положить какой-нибудь тяжелый предмет. Например, положим туда гирю массой 7 кг.

Теперь наоборот правая чаша перевесила левую. Экран по прежнему показывает, что чаши не равны.

Попробуем на левую чашу положить гирю массой 4 кг

Теперь весы выровнялись. На рисунке видно, что левая чаша на уровне правой чаши. А экран показывает знак равенства. Этот знак говорит о том, что левая чаша равна правой чаше.

Таким образом мы получили уравнение — равенство, содержащее неизвестное. Левая чаша — это левая часть уравнения, состоящая из компонентов 4 и переменной x (массы арбуза), а правая чаша — это правая часть уравнения, состоящая из компонента 7.

Ну и нетрудно догадаться, что корень уравнения 4 + x = 7 равен 3. Значит масса арбуза равна 3 кг.

Аналогично дела обстоят и с другими задачами. Чтобы найти какое-нибудь неизвестное значение, к левой или к правой части уравнения добавляют различные элементы: слагаемые, множители, выражения. В школьных задачах эти элементы бывают уже даны. Остается только правильно структурировать их и построить уравнение. Мы же в данном примере занимались подбором, пробуя гири разной массы, чтобы вычислить массу арбуза.

Естественно, те данные которые даны в задаче сначала нужно привести к виду, при котором их можно включить в уравнение. Поэтому, как говорят «хочешь не хочешь, а думать придётся».

Рассмотрим следующую задачу. Возраст отца равен возрасту сына и дочери вместе. Сын вдвое старше дочери и на двадцать лет моложе отца. Сколько лет каждому?

Возраст дочери можно обозначить через x . Если сын вдвое старше дочери, то его возраст будет обозначаться как 2x . В условии задачи сказано, что вместе возраст дочери и сына равен возрасту отца. Значит возраст отца будет обозначаться суммой x + 2x

В выражении можно привести подобные слагаемые. Тогда возраст отца будет обозначаться как 3x

Теперь составим уравнение. Нам нужно получить равенство в котором можно найти неизвестное x . Воспользуемся весами. На левую чашу положим возраст отца (3x) , а на правую чашу возраст сына (2x)

Понятно почему левая чаша перевесила правую и почему экран показывает знак ( ≠ ) . Ведь логично, что возраст отца больше возраста сына.

Но нам нужно уравнять весы, чтобы можно было вычислить неизвестное x . Для этого к правой чаше нужно прибавить какое-нибудь число. Какое именно число указано в задаче. В условии было сказано, что сын моложе отца на 20 лет. Значит 20 лет это то самое число, которое нужно положить на весы.

Весы выровнятся, если мы эти 20 лет добавим на правую чашу весов. Иными словами, вырастим сына до возраста отца

Теперь весы выровнялись. Получилось уравнение , которое решается легко:

В начале решения данной задачи через переменную x мы обозначили возраст дочери. Теперь мы нашли значение этой переменной. Дочери 20 лет.

Далее было сказано, что сын двое старше дочери, значит сыну (20 × 2) , то есть 40 лет.

Ну и наконец вычислим возраст отца. В задаче было сказано, что он равен сумме возрастов сына и дочери, то есть (20 + 40) лет.

Вернемся к середине задачи и обратим внимание на один момент. Когда мы положили на весы возраст отца и возраст сына, левая чаша перевесила правую

Но мы решили эту проблему, добавив на правую чашу еще 20 лет. В результате весы выровнялись и мы получили равенство

Но можно было не добавлять к правой чаше эти 20 лет, а вычесть их из левой. Мы получили бы равенство и в таком случае

В этот раз получается уравнение . Корень уравнения по прежнему равен 20

То есть уравнения и являются равносильными. А мы помним, что у равносильных уравнений корни совпадают. Если внимательно посмотреть на эти два уравнения, то можно увидеть что второе уравнение получено путем переноса числа 20 из правой части в левую с противоположным знаком. А это действие, как было указано в предыдущем уроке, не меняет корней уравнения.

Также нужно обратить внимание на то, что в начале решения задачи возрасты каждого члена семьи можно было обозначить через другие выражения.

Скажем возраст сына обозначить через x и поскольку он двое старше дочери, то возраст дочери обозначить через (понимай сделать её младше сына в два раза). А возраст отца поскольку он является суммой возрастов сына и дочери обозначить через выражение . Ну и напоследок для построения логически правильного уравнения, к возрасту сына нужно прибавить число 20, ведь отец старше на двадцать лет. В итоге получается совсем другое уравнение . Решим это уравнение

Как видно ответы к задаче не поменялись. Сыну по прежнему 40 лет. Дочери по прежнему лет, а отцу 40 + 20 лет.

Другими словами, задача может решаться различными методами. Поэтому не следует отчаиваться, что не получается решить ту или иную задачу. Но нужно иметь ввиду, что существует наиболее простые пути решения задачи. К центру города можно доехать различными маршрутами, но всегда существует наиболее удобный, быстрый и безопасный маршрут.

Примеры решения задач

Задача 1. В двух пачках всего 30 тетрадей. Если бы из первой пачки переложили во вторую 2 тетради, то в первой пачке стало бы вдвое больше тетрадей, чем во второй. Сколько тетрадей было в каждой пачке?

Решение

Обозначим через x количество тетрадей, которое было в первой пачке. Если всего тетрадей было 30, а переменная x это количество тетрадей из первой пачке, то количество тетрадей во второй пачке будет обозначаться через выражение 30 − x . То есть от общего количества тетрадей вычитаем количество тетрадей из первой пачки и тем самым получаем количество тетрадей из второй пачки.

Далее сказано, что если переложить 2 тетради из первой пачки во вторую, то в первой пачке окажется вдвое больше тетрадей. Итак, снимем с первой пачки две тетради

и добавим эти две тетради во вторую пачку

Выражения из которых мы будем составлять уравнение теперь принимают следующий вид:

Попробуем составить уравнение из имеющихся выражений. Положим на весы обе пачки тетрадей

Левая чаша тяжелее правой. Это потому, что в условии задачи сказано, что после того как из первой пачки взяли две тетради и положили их во вторую, количество тетрадей в первой пачке стало вдвое больше, чем во второй.

Чтобы выровнять весы и получить уравнение, увеличим правую часть вдвое. Для этого умножим её на 2

Получается уравнение . Решим данное уравнение:

Первую пачку мы обозначали через переменную x . Теперь мы нашли её значение. Переменная x равна 22. Значит в первой пачке было 22 тетради.

А вторую пачку мы обозначали через выражение 30 − x и поскольку значение переменой x теперь известно, то можно вычислить количество тетрадей во второй пачке. Оно равно 30 − 22 , то есть 8 шт .

Задача 2. Два человека чистили картофель. Один очищал в минуту две картофелины, а второй — три картофелины. Вместе они очистили 400 шт. Сколько времени работал каждый, если второй проработал на 25 минут больше первого?

Решение

Обозначим через x время работы первого человека. Поскольку второй человек проработал на 25 минут больше первого, то его время будет обозначаться через выражение

Первый рабочий в минуту очищал 2 картофелины, и поскольку он работал x минут, то всего он очистил 2x картофелин.

Второй человек в минуту очищал три картофелины, и поскольку он работал минут, то всего он очистил картофелин.

Вместе они очистили 400 картофелин

Из имеющихся компонентов составим и решим уравнение. В левой части уравнения будут картофелины, очищенные каждым человеком, а в правой части их сумма:

В начале решения данной задачи через переменную x мы обозначили время работы первого человека. Теперь мы нашли значение этой переменной. Первый человек работал 65 минут.

А второй человек работал минут, и поскольку значение переменной x теперь известно, то можно вычислить время работы второго человека — оно равно 65 + 25 , то есть 90 мин .

Задача из Учебника по алгебре Андрея Петровича Киселева. Из сортов чая составлена смесь в 32 кг. Килограмм первого сорта стоит 8 руб., а второго сорта 6 руб. 50 коп. Сколько килограммов взято того и другого сорта, если килограмм смеси стоит (без прибыли и убытка) 7 руб. 10 коп.?

Решение

Обозначим через x массу чая первого сорта. Тогда масса чая второго сорта будет обозначаться через выражение 32 − x

Килограмм чая первого сорта стоит 8 руб. Если эти восемь рублей умножить на количество килограмм чая первого сорта, то можно будет узнать во сколько рублей обошлись x кг чая первого сорта.

А килограмм чая второго сорта стоит 6 руб. 50 коп. Если эти 6 руб. 50 коп. умножить на 32 − x , то можно узнать во сколько рублей обошлись 32 − x кг чая второго сорта.

В условии сказано, что килограмм смеси стоит 7 руб. 10 коп. Всего же было приготовлено 32 кг смеси. Умножим 7 руб. 10 коп. на 32 мы сможем узнать сколько стоит 32 кг смеси.

Выражения из которых мы будем составлять уравнение теперь принимают следующий вид:

Попробуем составить уравнение из имеющихся выражений. Положим на левую чашу весов стоимость смесей чая первого и второго сорта, а на правую чашу положим стоимость 32 кг смеси, то есть общую стоимость смеси, в составе которой оба сорта чая:

Получили уравнение . Решим его:

В начале решения данной задачи через переменную x мы обозначили массу чая первого сорта. Теперь мы нашли значение этой переменной. Переменная x равна 12,8. Значит для приготовления смеси было взято 12,8 кг чая первого сорта.

А через выражение 32 − x мы обозначили массу чая второго сорта и поскольку значение переменой x теперь известно, то можно вычислить массу чая второго сорта. Оно равно 32 − 12,8 то есть 19,2 . Значит для приготовления смеси было взято 19,2 кг чая второго сорта.

Задача 3. Велосипедист проехал некоторое расстояние со скоростью 8 км/ч. Возвратиться он должен был другой дорогой, которая была на 3 км длиннее первой, и, хотя возвращаясь, ехал со скоростью 9 км/ч, он употребил времени на минут более. Как длинны были дороги?

Решение

Некоторые задачи могут затрагивать темы, которые человек возможно не изучал. Данная задача относится к такому кругу задач. В ней затрагиваются понятия расстояния, скорости и времени. Соответственно, чтобы решить подобную задачу, нужно иметь представление о тех вещах, о которых говорится в задаче. В нашем случае, надо знать что представляет собой расстояние, скорость и время.

В задаче нужно найти расстояния двух дорог. Мы должны составить уравнение, которое позволит вычислить эти расстояния.

Вспомним, как взаимосвязаны расстояние, скорость и время. Каждая из этих величин может быть описана с помощью буквенного уравнения:

Правую часть одного из этих уравнений мы будем использовать для составления своего уравнения. Чтобы узнать какую именно, нужно вернуться к тексту задачи и обратить внимание на следующий момент:

Следует обратить внимание на момент, где велосипедист на обратном пути употребил времени на минут более. Эта подсказка указывает нам, что можно воспользоваться уравнением , а именно его правой частью. Это позволит нам составить уравнение, которое содержит переменную S .

Итак, обозначим длину первой дороги через S . Этот путь велосипедист проехал со скоростью 8 км/ч . Время за которое он преодолел этот путь будет обозначаться выражением , поскольку время это отношение пройденного расстояния к скорости

Обратная дорога для велосипедиста была длиннее на 3 км . Поэтому её расстояние будет обозначаться через выражение S + 3 . Эту дорогу велосипедист проехал со скоростью 9 км/ч . А значит время за которое он преодолел этот путь будет обозначаться выражением .

Теперь составим уравнение из имеющихся выражений

Правая чаша тяжелее левой. Это потому, что в задаче сказано, что на обратную дорогу велосипедист затратил времени на больше.

Чтобы уравнять весы прибавим к левой части эти самые минут. Но сначала переведем минуты в часы, поскольку в задаче скорость измеряется в километрах в час, а не в метрах в минуту.

Чтобы минут перевести в часы, нужно разделить их на 60

минут составляют часа. Прибавляем эти часа к левой части уравнения:

Получается уравнение . Решим данное уравнение. Чтобы избавиться от дробей, обе части части можно умножить на 72. Далее пользуясь известными тождественными преобразованиями, найдем значение переменной S

Через переменную S мы обозначали расстояние первой дороги. Теперь мы нашли значение этой переменной. Переменная S равна 15. Значит расстояние первой дороги составляет 15 км.

А расстояние второй дороги мы обозначили через выражение S + 3 , и поскольку значение переменной S теперь известно, то можно вычислить расстояние второй дороги. Это расстояние равно сумме 15 + 3 , то есть 18 км .

Задача 4. По шоссе идут две машины с одной и той же скоростью. Если первая увеличит скорость на 10 км/ч, а вторая уменьшит скорость на 10 км/ч, то первая за 2 ч пройдет столько же, сколько вторая за 3 ч. С какой скоростью идут автомашины?

Решение

Обозначим через v скорость каждой машины. Далее в задаче приводятся подсказки: скорость первой машины увеличить на 10 км/ч, а скорость второй — уменьшить на 10 км/ч. Воспользуемся этой подсказкой

Далее говорится, что при таких скоростях (увеличенных и уменьшенных на 10 км/ч) первая машина пройдет за 2 часа столько же расстояния сколько вторая за 3 часа. Фразу «столько же» можно понимать как «расстояние, пройденное первой машиной, будет равно расстоянию, пройденному второй машиной».

Расстояние как мы помним, определяется по формуле . Нас интересует правая часть этого буквенного уравнения — она позволит нам составить уравнение, содержащее переменную v .

Итак, при скорости v + 10 км/ч первая машина пройдет 2(v+10) км , а вторая пройдет 3(v − 10) км . При таком условии машины пройдут одинаковые расстояния, поэтому для получения уравнения достаточно соединить эти два выражения знаком равенства. Тогда получим уравнение . Решим его:

В условии задачи было сказано, что машины идут с одинаковой скоростью. Мы обозначили эту скорость через переменную v . Теперь мы нашли значение этой переменной. Переменная v равна 50. Значит скорость обеих машин составляла 50 км/ч.

Задача 5. За 9 ч по течению реки теплоход проходит тот же путь, что за 11 ч против течения. Найдите собственную скорость теплохода, если скорость течения реки 2 км/ч.

Решение

Обозначим через v собственную скорость теплохода. Скорость течения реки равна 2 км/ч. По течению реки скорость теплохода будет составлять v + 2 км/ч , а против течения — (v − 2) км/ч .

В условии задачи сказано, что за 9 ч по течению реки теплоход проходит тот же путь, что за 11 ч против течения. Фразу «тот же путь» можно понимать как «расстояние, пройденное теплоходом по течению реки за 9 часов, равно расстоянию, пройденному теплоходом против течения реки за 11 часов». То есть расстояния будут одинаковыми.

Расстояние определяется по формуле . Воспользуемся правой частью этого буквенного уравнения для составления своего уравнения.

Итак, за 9 часов по течению реки теплоход пройдет 9(v + 2) км , а за 11 часов против течения — 11(v − 2) км . Поскольку оба выражения описывают одно и то же расстояние, приравняем первое выражение ко второму. В результате получим уравнение . Решим его:

Значит собственная скорость теплохода составляет 20 км/ч.

При решении задач полезной привычкой является заранее определить на каком множестве ищется для неё решение.

Допустим, что в задаче требовалось найти время, за которое пешеход преодолеет указанный путь. Мы обозначили время через переменную t , далее составили уравнение, содержащее эту переменную и нашли её значение.

Из практики мы знаем, что время движения объекта может принимать как целые значения, так и дробные, например 2 ч, 1,5 ч, 0,5 ч. Тогда можно сказать, что решение данной задачи ищется на множестве рациональных чисел Q, поскольку каждое из значений 2 ч, 1,5 ч, 0,5 ч может быть представлено в виде дроби.

Поэтому после того, как неизвестную величину обозначили через переменную, полезно указать к какому множеству эта величина принадлежит. В нашем примере время t принадлежит множеству рациональных чисел Q

Ещё можно ввести ограничение для переменной t , указав что она может принимать только положительные значения. Действительно, если объект затратил на путь определенное время, то это время не может быть отрицательным. Поэтому рядом с выражением tQ укажем, что её значение должно быть больше нуля:

Если решив уравнение, мы получим отрицательное значение для переменной t , то можно будет сделать вывод, что задача решена неправильно, поскольку это решение не будет удовлетворять условию tQ , t > 0 .

Ещё пример. Если бы мы решали задачу в которой требовалось найти количество человек для выполнения той или иной работы, то это количество мы обозначили бы через переменную x . В такой задаче решение искалось бы на множестве натуральных чисел

Действительно, количество человек является целым числом, например 2 человека, 3 человека, 5 человек. Но никак не 1,5 (один целый человек и половина человека) или 2,3 (два целых человека и еще три десятых человека).

Здесь можно было бы указать, что количество человек должно быть больше нуля, но числа входящие во множество натуральных чисел N сами по себе являются положительными и большими нуля. В этом множестве нет отрицательных чисел и числа 0. Поэтому выражение x > 0 можно не писать.

Задача 6. Для ремонта школы прибыла бригада в которой было в 2,5 раза больше маляров, чем плотников. Вскоре прораб включил в бригаду еще четырех маляров, а двух плотников перевел на другой объект. В результате маляров в бригаде оказалось в 4 раза больше чем плотников. Сколько маляров и сколько плотников было в бригаде первоначально

Решение

Обозначим через x плотников, прибывших на ремонт первоначально.

Количество плотников является целым числом, большим нуля. Поэтому укажем, что x принадлежит множество натуральных чисел

Маляров было в 2,5 раза больше, чем плотников. Поэтому количество маляров будет обозначаться как 2,5x .

Далее говорится, что прораб включил в бригаду еще четырех маляров, а двух плотников перевел на другой объект. Сделаем для своих выражений тоже самое. Уменьшим количество плотников на 2

А количество маляров увеличим на 4

Теперь количество плотников и маляров будут обозначаться через следующие выражения:

Попробуем составить уравнение из имеющихся выражений:

Правая чаша больше, поскольку после включения в бригаду ещё четырёх маляров, и перемещения двух плотников на другой объект, количество маляров в бригаде оказалось в 4 раза больше чем плотников. Чтобы уравнять весы, нужно левую чашу увеличить в 4 раза:

Получили уравнение . Решим его:

Через переменную x было обозначено первоначальное количество плотников. Теперь мы нашли значение этой переменной. Переменная x равна 8. Значит 8 плотников было в бригаде первоначально.

А количество маляров было обозначено через выражение 2,5 x и поскольку значение переменной x теперь известно, то можно вычислить количество маляров — оно равно 2,5 × 8 , то есть 20 .

Возвращаемся к началу задачи и удостоверяемся, что соблюдается условие xN. Переменная x равна 8, а элементы множества натуральных чисел N это все числа, начинающиеся с 1, 2, 3 и так далее до бесконечности. В это же множество входит число 8, которое мы нашли.

Тоже самое можно сказать о количестве маляров. Число 20 принадлежит множеству натуральных чисел:

Для понимания сути задачи и правильного составления уравнения, вовсе необязательно использовать модель весов с чашами. Можно использовать и другие модели: отрезки, таблицы, схемы. Можно придумать свою модель, которая хорошо описывала бы суть задачи.

Задача 9. Из бидона отлили 30% молока. В результате в нем осталось 14 л. Сколько литров молока было в бидоне первоначально?

Решение

Искомое значение это первоначальное число литров в бидоне. Изобразим число литров в виде линии и подпишем эту линию как X

Сказано, что из бидона отлили 30% молока. Выделим на рисунке приблизительно 30%

Процент по определению есть одна сотая часть чего-то. Если 30% молока отлили, то остальные 70% остались в бидоне. На эти 70% приходятся 14 литров, указанные в задаче. Выделим на рисунке оставшиеся 70%

Теперь можно составить уравнение. Вспомним, как находить процент от числа. Для этого общее количество чего-то делят на 100 и полученный результат умножают на искомое количество процентов. Замечаем, что 14 литров, составляющих 70% можно получить таким же образом: первоначальное число литров X разделить на 100 и полученный результат умножить на 70. Всё это приравнять к числу 14

Или получить более простое уравнение: 70% записать как 0,70, затем умножить на X и приравнять это выражение к 14

Значит первоначально в бидоне было 20 литров молока.

Задача 9. Взяли два сплава золота и серебра. В одном количество этих металлов находится в отношении 1 : 9, а в другом 2 : 3. Сколько нужно взять каждого сплава, чтобы получить 15 кг нового сплава, в котором золото и серебро относилось бы как 1 : 4?

Решение

Попробуем сначала узнать сколько золота и серебра будет содержáться в 15 кг нового сплава. В задаче сказано, что содержание этих металлов должно быть в отношении 1 : 4, то есть на одну часть сплава должно приходиться золото, а на четыре части — серебро. Тогда всего частей в сплаве будет 1 + 4 = 5, а масса одной части будет 15 : 5 = 3 кг.

Определим сколько золота будет содержáться в 15 кг сплава. Для этого 3 кг умножим на количество частей золота:

Определим сколько серебра будет содержáться в 15 кг сплава:

Значит сплав массой 15 кг будет содержать 3 кг золота и 12 кг серебра. Теперь вернёмся к исходным сплавам. Использовать нужно каждый из них. Обозначим через x массу первого сплава, а массу второго сплава можно обозначить через 15 − x

Выразим в процентах все отношения, которые даны в задаче и заполним ими следующую таблицу:

В первом сплаве золото и серебро находятся в отношении 1 : 9. Тогда всего частей будет 1 + 9 = 10 . Из них золота будет , а серебра .

Перенесём эти данные в таблицу. 10% занесём в первую строку в графу «процент золота в сплаве», 90% также занесём в первую строку графу «процент серебра в сплаве», а в последнюю графу «масса сплава» занесём переменную x , поскольку так мы обозначили массу первого сплава:

Аналогично поступаем со вторым сплавом. Золото и серебро в нём находятся в отношении 2 : 3. Тогда всего частей будет 2 + 3 = 5. Из них золота будет , а серебра .

Перенесём эти данные в таблицу. 40% занесем во вторую строку в графу «процент золота в сплаве», 60% также занесём во вторую строку графу «процент серебра в сплаве», а в последнюю графу «масса сплава» занесём выражение 15 − x , поскольку так мы обозначили массу второго сплава:

Заполним последнюю строку. Полученный сплав массой 15 кг будет содержать 3 кг золота, что составляет сплава, а серебра будет сплава. В последнюю графу записываем массу полученного сплава 15

Теперь по данной таблице можно составить уравнения. Вспоминаем задачи на концентрацию, сплавы и смеси. Если мы отдельно сложим золото обоих сплавов и приравняем эту сумму к массе золота полученного сплава, то сможем узнать чему равно значение x.

Далее для удобства проценты будем выражать в десятичной дроби.

В первом сплаве золота было 0,10x , а во втором сплаве золота было 0,40(15 − x) . Тогда в полученном сплаве масса золота будет суммой масс золота первого и второго сплавов и эта масса составляет 20% от нового сплава. А 20% от нового сплава это 3 кг золота, вычисленные нами ранее. В результате получаем уравнение 0,10x + 0.40(15 − x) = 3 . Решим это уравнение:

Изначально через x мы обозначили массу первого сплава. Теперь мы нашли значение этой переменной. Переменная x равна 10. А массу второго сплава мы обозначили через 15 − x , и поскольку значение переменной x теперь известно, то можно вычислить массу второго сплава, она равна 15 − 10 = 5 кг .

Значит для получения нового сплава массой 15 кг в котором золото и серебро относились бы как 1 : 4, нужно взять 10 кг первого и 5 кг второго сплава.

Уравнение можно было составить, воспользовавшись и вторым столбцом получившейся таблицы. Тогда мы получили бы уравнение 0,90x + 0.60(15 − x) = 12. Корень этого уравнения тоже равен 10

Задача 10. Имеется руда из двух пластов с содержанием меди в 6% и 11%. Сколько надо взять бедной руды, чтобы получить при смешивании с богатой 20 тонн с содержанием меди 8%?

Решение

Обозначим через x массу бедной руды. Поскольку нужно получить 20 тонн руды, то богатой руды будет взято 20 − x . Поскольку содержание меди в бедной руде составляет 6%, то в x тоннах руды будет содержáться 0,06x тонн меди. В богатой руде содержание меди составляет 11%, а в 20 − x тоннах богатой руды будет содержáться 0,11(20 − x) тонн меди.

В получившихся 20 тоннах руды содержание меди должно составлять 8%. Значит в 20 тоннах руды меди будет содержáться 20 × 0,08 = 1,6 тонн.

Сложим выражения 0,06x и 0,11(20 − x) и приравняем эту сумму к 1,6. Получим уравнение 0,06x + 0,11(20 − x) = 1,6

Решим данное уравнение:

Значит для получения 20 тонн руды с содержанием меди 8%, нужно взять 12 тонн бедной руды. Богатой же будет взято 20 − 12 = 8 тонн.

Задача 11. Увеличив среднюю скорость с 250 до 300 м/мин спортсменка стала пробегать дистанцию на 1 мин быстрее. Какова длина дистанции?

Решение

Длину дистанции (или расстояние дистанции) можно описать следующим буквенным уравнением:

Воспользуемся правой частью этого уравнения для составления своего уравнения. Изначально спортсменка пробегала дистанцию со скоростью 250 метров в минуту. При такой скорости длина дистанции будет описываться выражением 250t

Затем спортсменка увеличила свою скорость до 300 метров в минуту. При такой скорости длина дистанции будет описываться выражением 300t

Заметим, что длина дистанции это величина постоянная. От того, что спортсменка увеличит скорость или уменьшит её, длина дистанции останется неизменной.

Это позволяет нам приравнять выражение 250t к выражению 300t , поскольку оба выражения описывают длину одной и той же дистанции

Но в задаче сказано, что при скорости 300 метров в минуту спортсменка стала пробегать дистанцию на 1 минуту быстрее. Другими словами, при скорости 300 метров в минуту, время движения уменьшится на единицу. Поэтому в уравнении 250t = 300t в правой части время нужно уменьшить на единицу:

Получилось простейшее уравнение. Решим его:

При скорости 250 метров в минуту спортсменка пробегает дистанцию за 6 минут. Зная скорость и время, можно определить длину дистанции:

S = 250 × 6 = 1500 м

А при скорости 300 метров в минуту спортсменка пробегает дистанцию за t − 1 , то есть за 5 минут. Как было сказано ранее длина дистанции не меняется:

S = 300 × 5 = 1500 м

Задача 12. Всадник догоняет пешехода, находящегося впереди него на 15 км. Через сколько часов всадник догонит пешехода, если каждый час первый проезжает по 10 км, а второй проходит только по 4 км?

Решение

Данная задача является задачей на движение. Её можно решить, определив скорость сближения и разделив изначальное расстояние между всадником и пешеходом на эту скорость.

Скорость сближения определяется вычитанием меньшей скорости из большей:

10 км/ч − 4 км/ч = 6 км/ч (скорость сближения)

С каждым часом расстояние в 15 километров будут сокращаться на 6 км. Чтобы узнать, когда оно сократится полностью (когда всадник догонит пешехода), нужно 15 разделить на 6

2,5 ч это два целых часа и половина часа. А половина часа это 30 минут. Значит всадник догонит пешехода через 2 часа 30 минут.

Решим эту задачу с помощью уравнения.

Будем считать, что пешеход и всадник вышли в путь из одного и того же места. Пешеход вышел раньше всадника и успел преодолеть 15 км

После этого вслед за ним в путь вышел всадник со скоростью 10 км/ч. А скорость пешехода составляет только 4 км/ч. Это значит, что всадник через некоторое время догонит пешехода. Это время нам нужно найти.

Когда всадник догонит пешехода это будет означать, что они вместе прошли одинаковое расстояние. Расстояние, пройденное всадником и пешеходом описывается следующим уравнением:

Воспользуемся правой частью этого уравнения для составления своего уравнения.

Расстояние, пройденное всадником, будет описываться выражением 10t . Поскольку пешеход вышел в путь раньше всадника и успел преодолеть 15 км, то расстояние пройденное им будет описываться выражением 4t + 15 .

На момент, когда всадник догонит пешехода, оба они пройдут одинаковое расстояние. Это позволяет нам приравнять расстояния, пройденные всадником и пешеходом:

Получилось простейшее уравнение. Решим его:

Задачи для самостоятельного решения

Решение

Скорости поездов в данной задаче измеряются в километрах в час. Поэтому 45 мин, указанные в задаче, переведем в часы. 45 мин это 0,75 ч

Обозначим время, за которое товарный поезд приезжает в город, через переменную t . Поскольку пассажирский поезд приезжает в этот город на 0,75 ч быстрее, то время его движения будет обозначаться через выражение t − 0,75

Пассажирский поезд преодолел 48(t − 0.75) км, а товарный 36t км. Поскольку речь идет об одном и том же расстоянии, приравняем первое выражение ко второму. В результате получим уравнение 48(t − 0.75) = 36t . Решим его:

Теперь вычислим расстояние между городами. Для этого скорость товарного поезда (36 км/ч) умножим на время его движения t. Значение переменной t теперь известно — оно равно трём часам

Для вычисления расстояния можно воспользоваться и скоростью пассажирского поезда. Но в этом случае значение переменной t необходимо уменьшить на 0,75 поскольку пассажирский поезд затратил времени на 0,75 ч меньше

48 × (3 − 0,75) = 144 − 36 = 108 км

Ответ: расстояние между городами равно 108 км.

Решение

Пусть t время через которое автомобили встретились. Тогда первый автомобиль на момент встречи проедет 65t км, а второй 60t км. Сложим эти расстояния и приравняем к 150. Получим уравнение 65t + 60t = 150

Значение переменной t равно 1,2. Значит автомобили встретились через 1,2 часа.

Ответ: автомобили встретились через 1,2 часа.

Решение

Пусть x рабочих было в первом цехе. Во втором цехе было в три раза больше, чем в первом, поэтому количество рабочих во втором цехе можно обозначить через выражение 3x . В третьем цехе было на 15 рабочих меньше, чем во втором. Поэтому количество рабочих в третьем цехе можно обозначить через выражение 3x − 15 .

В задаче сказано, что всего рабочих было 685. Поэтому можно сложить выражения x, 3x, 3x − 15 и приравнять эту сумму к числу 685. В результате получим уравнение x + 3x + (3x − 15) = 685

Через переменную x было обозначено количество рабочих в первом цехе. Теперь мы нашли значение этой переменной, оно равно 100. Значит в первом цехе было 100 рабочих.

Во втором цехе было 3x рабочих, то есть 3 × 100 = 300 . А в третьем цехе было 3x − 15 , то есть 3 × 100 − 15 = 285

Ответ: в первом цехе было 100 рабочих, во втором — 300, в третьем — 285.

Решение

Пусть x моторов должна была отремонтировать первая мастерская. Тогда вторая мастерская должна была отремонтировать 18 − x моторов .

Поскольку первая мастерская выполнила свой план на 120%, это означает что она отремонтировала 1,2x моторов . А вторая мастерская выполнила свой план на 125%, значит она отремонтировала 1,25(18 − x) моторов.

В задаче сказано, что было отремонтировано 22 мотора. Поэтому можно сложить выражения 1,2x и 1,25(18 − x) , затем приравнять эту сумму к числу 22. В результате получим уравнение 1,2x + 1,25(18 − x) = 22

Через переменную x было обозначено количество моторов, которые должна была отремонтировать первая мастерская. Теперь мы нашли значение этой переменной, она равна 10. Значит первая мастерская должна была отремонтировать 10 моторов.

А через выражение 18 − x было обозначено количество моторов, которые должна была отремонтировать вторая мастерская. Значит вторая мастерская должна была отремонтировать 18 − 10 = 8 моторов.

Ответ: первая мастерская должна была отремонтировать 10 моторов, а вторая — 8 моторов.

Решение

Пусть x рублей стоил товар до повышения цены. Если цена увеличилась на 30% это означает, что она увеличилась на 0,30x рублей. После повышения цены товар начал стоить 91 руб. Сложим x с 0,30x и приравняем эту сумму к 91. В результате получим уравнение x + 0.30x = 91

Значит до повышения цены товар стоил 70 рублей.

Ответ: до повышения цены товар стоил 70 рублей.

Решение

Пусть x — исходное число. Увеличим его на 25%. Получим выражение x + 0,25x . Приведем подобные слагаемые, получим x + 0,25x = 1.25x .

Узнаем какую часть исходное число x составляет от нового числа 1,25x

Если новое число 1,25x считать за 100%, а исходное число x составляет от него 80%, то уменьшив новое число на 20% можно получить исходное число x

Ответ: чтобы получить исходное число, новое число нужно уменьшить на 20%.

Решение

Пусть x — первоначальное число. Увеличим его на 20%. Получим выражение x + 0,20x . Приравняем эту сумму к числу 144, получим уравнение x + 0,20x = 144

Ответ: первоначальное значение числа равно 120.

Решение

Пусть x — первоначальное число. Уменьшим его на 10%. Получим выражение x − 0,10x . Приравняем эту разность к числу 45, получим уравнение x − 0,10x = 45

Ответ: первоначальное значение числа равно 50.

Решение

Пусть x рублей — первоначальная цена альбома. Снизим эту цену на 15%, получим x − 0,15x . Снизим цену ещё на 15 руб., получим x − 0,15x − 15 . После этих снижений альбом стал стоить 19 руб. Приравняем выражение x − 0,15x − 15 к числу 19, получим уравнение x − 0,15x − 15 = 19

Ответ: первоначальная цена альбома составляет 40 руб.

Решение

Если 80% массы теряется, то на оставшиеся 20% будут приходиться 4 т сена. Пусть x тонн травы требуется для получения 4 т сена. Если 4 т будут составлять 20% травы, то можно составить уравнение:

Ответ: для получения 4 т сена, нужно накосить 20 т травы.

Решение

Пусть x кг 20%-го раствора соли нужно добавить к 1 кг 10%-го раствора.

В 1 кг 10%-го раствора соли содержится 0,1 кг соли. А в x кг 20%-го раствора соли содержится 0,20 x кг соли.

После добавления x кг 20%-го раствора в новом растворе будет содержáться 0,12(1 + x) кг соли. Сложим выражения 0,1 и 0,20x , затем приравняем эту сумму к выражению 0,12(1 + x) . В результате получим уравнение 0,1 + 0,20x = 0,12(1 + x)

Ответ: чтобы получить 12%-й раствор соли, нужно к 1 кг 10%-го раствора добавить 0,25 кг 20%-го раствора.

Решение

Пусть x кг первого раствора нужно взять. Поскольку требуется приготовить 25 кг раствора, то массу второго раствора можно обозначить через выражение 25 − x.

В первом растворе будет содержáться 0,20x кг соли, а втором — 0,30(25 − x) кг соли. В полученном растворе содержание соли будет 25 × 0,252 = 6,3 кг. Сложим выражения 0,20x и 0,30(25 − x), затем приравняем эту сумму к 6,3. В результате получим уравнение

Значит первого раствора нужно взять 12 кг, а второго 25 − 12 = 13 кг.

Ответ: первого раствора нужно взять 12 кг, а второго 13 кг.

Понравился урок?
Вступай в нашу новую группу Вконтакте и начни получать уведомления о новых уроках

Возникло желание поддержать проект?
Используй кнопку ниже

44 thoughts on “Решение задач с помощью уравнений”

Вау новый урок. Я рад что вернулись)) После работы обязательно буду учить этот урок.

не смог решить ни одной задачи из примеров решения…

источники:

Умение решать уравнения необходимо для того, чтобы решать какие-то практические задачи по математике, физике, механике, экономике и другим предметам.

Пример:

в одном баке воды было в (4) раза больше, чем в другом. Из первого бака перелили в другой (36) литров и воды в баках стало поровну. Сколько литров воды было в каждом баке?

Решение:

сначала введём переменную, с помощью которой обозначим неизвестную нам величину, которую необходимо найти по условию задачи.

Пусть (x) л — количество воды, которое было до переливания во втором баке.

Тогда в первом баке её было (4x) л.

После переливания в первом баке осталось ((4x) (– 36)) л воды, а во втором стало ((x + 36)) л.

По условию задачи известно, что после переливания в обоих баках воды стало поровну. Составим уравнение:

(4x) (– 36 = x + 36).

Эту часть рассуждений при решении задач называют составлением математической модели.

На этом этапе текст задачи переводится с обычного языка на математический язык.

Математической моделью является составленное уравнение.

Затем начинается второй этап, называемый работой с математической моделью.

Здесь решается составленное уравнение:

4x−36=x+36;4x−x=36+36;3x=72;x=24.

Решив уравнение, переходим к третьему этапуответу на вопрос задачи.

Решив уравнение, получили (x=24), а за (x) принято количество воды в литрах, которое было до переливания во втором баке.

Значит, во втором баке было (24) л воды. По условию задачи в первом баке было в (4) раза больше воды, чем во втором. Значит, в первом баке было:

(24·4=96) (л).

Ответ: в одном баке было (24) л воды, а в другом баке было (96) л воды.

Таким образом, в ходе решения было выделено три этапа математического моделирования:

1) составление математической модели (составление уравнения по условию задачи);

2) работа с математической моделью (решение уравнения);

3) ответ на вопрос задачи.

Для составления математической модели нужно провести анализ задачи, результаты которого можно оформить в виде таблицы, схемы, рисунка, краткой записи.

Правила составления уравнений в математике

Составить уравнение — значит выразить в математической форме связь между данными (известными) задачи и искомыми (неизвестными) ее величинами. Иногда эта связь, настолько явно содержится в формулировке задачи, что составление уравнения есть просто дословный пересказ задачи, на языке математических знаков.

Пример 1

Петров получил за работу на 160 руб. больше, чем половина суммы, которую получил Иванов. Вместе они получили 1120 руб. Сколько получили за работу Петров и Иванов?

Обозначим через х заработок Иванова. Половина его заработка есть 0,5x ; месячной заработок Петрова 0,5x + 160 вместе они зарабатывают 1120 руб.; математическая запись последней фразы будет

( 0,5x + 160 ) + x = 1120.

Уравнение составлено. Решая его по раз установленным правилам, находим, заработок Иванова х = 640руб.; заработок же Петрова 0,5x + 160 = 480 (руб.).

Чаще, однако, случается, что связь между данными и искомыми величинами не указывается в задаче прямо; ее нужно установить, исходя из условий задачи. В практических задачах так и бывает почти всегда. Только что приведенный пример носит надуманный характер; в жизни почти никогда подобных задач не встречается.

Для составления уравнения поэтому нельзя дать вполне исчерпывающих указаний. Однако на первых порах полезно руководствоваться следующим. Примем за значение искомой величины (или нескольких величин) какое-нибудь наугад взятое число (или несколько чисел) и поставим себе задачу проверить, угадали ли мы правильное решение задачи или нет. Если мы сумели провести эту проверку и обнаружить либо то, что догадка наша верна, либо то, что она неверна (скорее всего случится, конечно, второе), то мы немедленно можем составить нужное уравнение (или несколько уравнений). Именно, запишем те самые действия, которые мы производили для проверки, только вместо наугад взятого числа введем буквенной знак неизвестной величины. Мы получим требуемое уравнение.

Пример 1

Кусок сплава меди и цинка объемом в 1 дм 3 весит 8,14 кг. Сколько меди содержится в сплаве? (уд. вес меди 8,9 кг/дм 3 ; цинка — 7,0 кг/дм 3 ).

Возьмем наугад число, выражающее искомый объем меди, например 0,3 дм 3 . Проверим, удачно ли мы взяли это число. Так как 1 кг/дм 3 меди весит 8,9 кг, то 0,3 дм 3 весят 8,9 * 0,3 = 2,67 (кг). Объем цинка в сплаве есть 1 — 0,3 = 0,7 (дм 3 ). Вес его 7,0 * 0,7 = 4,9 (кг). Общий вес цинка и меди 2,67+ +4,9 = 7,57 (кг). Между тем вес нашего куска, по условию задачи, 8,14 кг. Догадка наша несостоятельна. Но зато мы немедленно получим уравнение решение которого даст правильный ответ. Вместо наугад взятого числа 0,3 дм 3 обозначим объем меди (в дм 3 ) через х. Вместо произведения 8,9 * 0,3 = 2,67 берем произведшие 8,9 x. Это — вес меди в сплаве. Вместо 1 – 0,3 = 0,7 берем 1 — х; это — объем цинка. Вместо 7,0 * 0,7 = 4,9 берем 7,0 (1 — x); это — вес цинка. Вместо 2,67 + 4,9 берем 8,9х + 7,0 (1 — х); это — общий вес цинка и меди. По условию он равен 8,14 кг; значит, 8,9х +7,0 (1 — x) = 8,14.

Решение этого уравнения дает x = 0,6. Проверку наугад взятого решения можно делать различными способами; соответственно этому можно получить для одной и той же задачи различные виды уравнения; все они, однако, дадут для искомой величины одно и, то же решение, такие уравнения называются равносильными друг другу.

Разумеется, после получения навыков в составлении уравнений нет нужды производить проверку наугад взятого числа: можно для значения искомой величины брать не число, а какую-нибудь букву (х, у и т. д.) и поступать так, как если бы эта буква (неизвестное) была тем числом, проверить которое мы собираемся.

Решение простых линейных уравнений

О чем эта статья:

Статья находится на проверке у методистов Skysmart.
Если вы заметили ошибку, сообщите об этом в онлайн-чат
(в правом нижнем углу экрана).

Понятие уравнения

Уравнение — это математическое равенство, в котором неизвестна одна или несколько величин. Значение неизвестных нужно найти так, чтобы при их подстановке в пример получилось верное числовое равенство.

Например, возьмем выражение 2 + 4 = 6. При вычислении левой части получается верное числовое равенство, то есть 6 = 6.

Уравнением можно назвать выражение 2 + x = 6, с неизвестной переменной x, значение которой нужно найти. Результат должен быть таким, чтобы знак равенства был оправдан, и левая часть равнялась правой.

Корень уравнения — то самое число, которое при подстановке на место неизвестной уравнивает выражения справа и слева.

Решить уравнение значит найти все возможные корни или убедиться, что их нет.

Решить уравнение с двумя, тремя и более переменными — это два, три и более значения переменных, которые обращают данное выражение в верное числовое равенство.

Равносильные уравнения — это те, в которых совпадают множества решений. Другими словами, у них одни и те же корни.

Какие бывают виды уравнений

Уравнения могут быть разными, самые часто встречающиеся — линейные и квадратные.

Особенность преобразований алгебраических уравнений в том, что в левой части должен остаться многочлен от неизвестных, а в правой — нуль.

Линейное уравнение выглядит так ах + b = 0, где a и b — действительные числа.

Что поможет в решении:

  • если а не равно нулю, то у уравнения единственный корень: х = -b : а;
  • если а равно нулю — у уравнения нет корней;
  • если а и b равны нулю, то корень уравнения — любое число.
Квадратное уравнение выглядит так: ax 2 + bx + c = 0, где коэффициенты a, b и c — произвольные числа, a ≠ 0.

Числовой коэффициент — число, которое стоит при неизвестной переменной.

Кроме линейных и квадратных есть и другие виды уравнений, с которыми мы познакомимся в следующий раз:

Онлайн-курсы по математике за 7 класс помогут закрепить новые знания на практике с талантливым преподавателем.

Как решать простые уравнения

Чтобы научиться решать простые линейные уравнения, нужно запомнить формулу и два основных правила.

1. Правило переноса. При переносе из одной части в другую, член уравнения меняет свой знак на противоположный.

Для примера рассмотрим простейшее уравнение: x+3=5

Начнем с того, что в каждом уравнении есть левая и правая часть.

Перенесем 3 из левой части в правую и меняем знак на противоположный.

Можно проверить: 2 + 3 = 5. Все верно. Корень равен 2.

Решим еще один пример: 6x = 5x + 10.

Перенесем 5x из правой части в левую. Знак меняем на противоположный, то есть на минус.

Приведем подобные и завершим решение.

2. Правило деления. В любом уравнении можно разделить левую и правую часть на одно и то же число. Это может ускорить процесс решения. Главное — быть внимательным, чтобы не допустить глупых ошибок.

Применим правило при решении примера: 4x=8.

При неизвестной х стоит числовой коэффициент — 4. Их объединяет действие — умножение.

Чтобы решить уравнение, нужно сделать так, чтобы при неизвестной x стояла единица.

Разделим каждую часть на 4. Как это выглядит:

Теперь сократим дроби, которые у нас получились и завершим решение линейного уравнения:

Рассмотрим пример, когда неизвестная переменная стоит со знаком минус: −4x = 12

    Разделим обе части на −4, чтобы коэффициент при неизвестной стал равен единице.

−4x = 12 | : (−4)
x = −3

Если знак минус стоит перед скобками, и по ходу вычислений его убрали — важно не забыть поменять знаки внутри скобок на противоположные. Этот простой факт позволит не допустить обидные ошибки, особенно в старших классах.

Напомним, что не у каждого линейного уравнения есть решение — иногда корней просто нет. Изредка среди корней может оказаться ноль — ничего страшного, это не значит, что ход решения оказался неправильным. Ноль — такое же число, как и остальные.

Способов решения линейных уравнений немного, нужно запомнить только один алгоритм, который будет эффективен для любой задачки.

Алгоритм решения простого линейного уравнения
  1. Раскрываем скобки, если они есть.
  2. Группируем члены, которые содержат неизвестную переменную в одну часть уравнения, остальные члены — в другую.
  3. Приводим подобные члены в каждой части уравнения.
  4. Решаем уравнение, которое получилось: aх = b. Делим обе части на коэффициент при неизвестном.

Чтобы быстрее запомнить ход решения и формулу линейного уравнения, скачайте или распечатайте алгоритм — храните его в телефоне, учебнике или на рабочем столе.

Примеры линейных уравнений

Теперь мы знаем, как решать линейные уравнения. Осталось попрактиковаться на задачках, чтобы чувствовать себя увереннее на контрольных. Давайте решать вместе!

Пример 1. Как правильно решить уравнение: 6х + 1 = 19.

    Перенести 1 из левой части в правую со знаком минус.

Разделить обе части на множитель, стоящий перед переменной х, то есть на 6.

Пример 2. Как решить уравнение: 5(х − 3) + 2 = 3 (х − 4) + 2х − 1.

5х − 15 + 2 = 3х − 12 + 2х − 1

Сгруппировать в левой части члены с неизвестными, а в правой — свободные члены. Не забываем при переносе из одной части уравнения в другую поменять знаки на противоположные у переносимых членов.

5х − 3х − 2х = −12 − 1 + 15 − 2

Приведем подобные члены.

Ответ: х — любое число.

Пример 3. Решить: 4х = 1/8.

    Разделим обе части уравнения на множитель стоящий перед переменной х, то есть на 4.

Пример 4. Решить: 4(х + 2) = 6 − 7х.

  1. 4х + 8 = 6 − 7х
  2. 4х + 7х = 6 − 8
  3. 11х = −2
  4. х = −2 : 11
  5. х = −2/11

Ответ: −2/11 или −(0,18). О десятичных дробях можно почитать в другой нашей статье.

Пример 5. Решить:

  1. 3(3х — 4) = 4 · 7х + 24
  2. 9х — 12 = 28х + 24
  3. 9х — 28х = 24 + 12
  4. -19х = 36
  5. х = 36 : (-19)
  6. х = — 36/19

Пример 6. Как решить линейное уравнение: х + 7 = х + 4.

5х — 15 + 2 = 3х — 2 + 2х — 1

Сгруппировать в левой части неизвестные члены, в правой — свободные члены:

Приведем подобные члены.

Ответ: нет решений.

Пример 7. Решить: 2(х + 3) = 5 − 7х.

Общие сведения об уравнениях

Уравнения — одна из сложных тем для усвоения, но при этом они являются достаточно мощным инструментом для решения большинства задач.

С помощью уравнений описываются различные процессы, протекающие в природе. Уравнения широко применяются в других науках: в экономике, физике, биологии и химии.

В данном уроке мы попробуем понять суть простейших уравнений, научимся выражать неизвестные и решим несколько уравнений. По мере усвоения новых материалов, уравнения будут усложняться, поэтому понять основы очень важно.

Что такое уравнение?

Уравнение — это равенство, содержащее в себе переменную, значение которой требуется найти. Это значение должно быть таким, чтобы при его подстановке в исходное уравнение получалось верное числовое равенство.

Например выражение 3 + 2 = 5 является равенством. При вычислении левой части получается верное числовое равенство 5 = 5 .

А вот равенство 3 + x = 5 является уравнением, поскольку содержит в себе переменную x , значение которой можно найти. Значение должно быть таким, чтобы при подстановке этого значения в исходное уравнение, получилось верное числовое равенство.

Другими словами, мы должны найти такое значение, при котором знак равенства оправдал бы свое местоположение — левая часть должна быть равна правой части.

Уравнение 3 + x = 5 является элементарным. Значение переменной x равно числу 2. При любом другом значении равенство соблюдáться не будет

Говорят, что число 2 является корнем или решением уравнения 3 + x = 5

Корень или решение уравнения — это значение переменной, при котором уравнение обращается в верное числовое равенство.

Корней может быть несколько или не быть совсем. Решить уравнение означает найти его корни или доказать, что корней нет.

Переменную, входящую в уравнение, иначе называют неизвестным. Вы вправе называть как вам удобнее. Это синонимы.

Примечание. Словосочетание «решить уравнение» говорит самó за себя. Решить уравнение означает «уравнять» равенство — сделать его сбалансированным, чтобы левая часть равнялась правой части.

Выразить одно через другое

Изучение уравнений по традиции начинается с того, чтобы научиться выражать одно число, входящее в равенство, через ряд других. Давайте не будем нарушать эту традицию и поступим также.

Рассмотрим следующее выражение:

Данное выражение является суммой чисел 8 и 2. Значение данного выражения равно 10

Получили равенство. Теперь можно выразить любое число из этого равенства через другие числа, входящие в это же равенство. К примеру, выразим число 2.

Чтобы выразить число 2, нужно задать вопрос: «что нужно сделать с числами 10 и 8, чтобы получить число 2». Понятно, что для получения числа 2, нужно из числа 10 вычесть число 8.

Так и делаем. Записываем число 2 и через знак равенства говорим, что для получения этого числа 2 мы из числа 10 вычли число 8:

Мы выразили число 2 из равенства 8 + 2 = 10 . Как видно из примера, ничего сложного в этом нет.

При решении уравнений, в частности при выражении одного числа через другие, знак равенства удобно заменять на слово «есть». Делать это нужно мысленно, а не в самом выражении.

Так, выражая число 2 из равенства 8 + 2 = 10 мы получили равенство 2 = 10 − 8 . Данное равенство можно прочесть так:

2 есть 10 − 8

То есть знак = заменен на слово «есть». Более того, равенство 2 = 10 − 8 можно перевести с математического языка на полноценный человеческий язык. Тогда его можно будет прочитать следующим образом:

Число 2 есть разность числа 10 и числа 8

Число 2 есть разница между числом 10 и числом 8.

Но мы ограничимся лишь заменой знака равенства на слово «есть», и то будем делать это не всегда. Элементарные выражения можно понимать и без перевода математического языка на язык человеческий.

Вернём получившееся равенство 2 = 10 − 8 в первоначальное состояние:

Выразим в этот раз число 8. Что нужно сделать с остальными числами, чтобы получить число 8? Верно, нужно из числа 10 вычесть число 2

Вернем получившееся равенство 8 = 10 − 2 в первоначальное состояние:

В этот раз выразим число 10. Но оказывается, что десятку выражать не нужно, поскольку она уже выражена. Достаточно поменять местами левую и правую часть, тогда получится то, что нам нужно:

Пример 2. Рассмотрим равенство 8 − 2 = 6

Выразим из этого равенства число 8. Чтобы выразить число 8 остальные два числа нужно сложить:

Вернем получившееся равенство 8 = 6 + 2 в первоначальное состояние:

Выразим из этого равенства число 2. Чтобы выразить число 2, нужно из 8 вычесть 6

Пример 3. Рассмотрим равенство 3 × 2 = 6

Выразим число 3. Чтобы выразить число 3, нужно 6 разделить 2

Вернем получившееся равенство в первоначальное состояние:

Выразим из этого равенства число 2. Чтобы выразить число 2, нужно 6 разделить 3

Пример 4. Рассмотрим равенство

Выразим из этого равенства число 15. Чтобы выразить число 15, нужно перемножить числа 3 и 5

Вернем получившееся равенство 15 = 3 × 5 в первоначальное состояние:

Выразим из этого равенства число 5. Чтобы выразить число 5, нужно 15 разделить 3

Правила нахождения неизвестных

Рассмотрим несколько правил нахождения неизвестных. Возможно, они вам знакомы, но не мешает повторить их ещё раз. В дальнейшем их можно будет забыть, поскольку мы научимся решать уравнения, не применяя эти правила.

Вернемся к первому примеру, который мы рассматривали в предыдущей теме, где в равенстве 8 + 2 = 10 требовалось выразить число 2.

В равенстве 8 + 2 = 10 числа 8 и 2 являются слагаемыми, а число 10 — суммой.

Чтобы выразить число 2, мы поступили следующим образом:

То есть из суммы 10 вычли слагаемое 8.

Теперь представим, что в равенстве 8 + 2 = 10 вместо числа 2 располагается переменная x

В этом случае равенство 8 + 2 = 10 превращается в уравнение 8 + x = 10 , а переменная x берет на себя роль так называемого неизвестного слагаемого

Наша задача найти это неизвестное слагаемое, то есть решить уравнение 8 + x = 10 . Для нахождения неизвестного слагаемого предусмотрено следующее правило:

Чтобы найти неизвестное слагаемое, нужно из суммы вычесть известное слагаемое.

Что мы в принципе и сделали, когда выражали двойку в равенстве 8 + 2 = 10 . Чтобы выразить слагаемое 2, мы из суммы 10 вычли другое слагаемое 8

А сейчас, чтобы найти неизвестное слагаемое x , мы должны из суммы 10 вычесть известное слагаемое 8:

Если вычислить правую часть получившегося равенства, то можно узнать чему равна переменная x

Мы решили уравнение. Значение переменной x равно 2 . Для проверки значение переменной x отправляют в исходное уравнение 8 + x = 10 и подставляют вместо x. Так желательно поступать с любым решённым уравнением, поскольку нельзя быть точно уверенным, что уравнение решено правильно:

В результате получается верное числовое равенство. Значит уравнение решено правильно.

Это же правило действовало бы в случае, если неизвестным слагаемым было бы первое число 8.

В этом уравнении x — это неизвестное слагаемое, 2 — известное слагаемое, 10 — сумма. Чтобы найти неизвестное слагаемое x , нужно из суммы 10 вычесть известное слагаемое 2

Вернемся ко второму примеру из предыдущей темы, где в равенстве 8 − 2 = 6 требовалось выразить число 8.

В равенстве 8 − 2 = 6 число 8 это уменьшаемое, число 2 — вычитаемое, число 6 — разность

Чтобы выразить число 8, мы поступили следующим образом:

То есть сложили разность 6 и вычитаемое 2.

Теперь представим, что в равенстве 8 − 2 = 6 вместо числа 8 располагается переменная x

В этом случае переменная x берет на себя роль так называемого неизвестного уменьшаемого

Для нахождения неизвестного уменьшаемого предусмотрено следующее правило:

Чтобы найти неизвестное уменьшаемое, нужно к разности прибавить вычитаемое.

Что мы и сделали, когда выражали число 8 в равенстве 8 − 2 = 6 . Чтобы выразить уменьшаемое 8, мы к разности 6 прибавили вычитаемое 2.

А сейчас, чтобы найти неизвестное уменьшаемое x , мы должны к разности 6 прибавить вычитаемое 2

Если вычислить правую часть, то можно узнать чему равна переменная x

Теперь представим, что в равенстве 8 − 2 = 6 вместо числа 2 располагается переменная x

В этом случае переменная x берет на себя роль неизвестного вычитаемого

Для нахождения неизвестного вычитаемого предусмотрено следующее правило:

Чтобы найти неизвестное вычитаемое, нужно из уменьшаемого вычесть разность.

Что мы и сделали, когда выражали число 2 в равенстве 8 − 2 = 6. Чтобы выразить число 2, мы из уменьшаемого 8 вычли разность 6.

А сейчас, чтобы найти неизвестное вычитаемое x, нужно опять же из уменьшаемого 8 вычесть разность 6

Вычисляем правую часть и находим значение x

Вернемся к третьему примеру из предыдущей темы, где в равенстве 3 × 2 = 6 мы пробовали выразить число 3.

В равенстве 3 × 2 = 6 число 3 — это множимое, число 2 — множитель, число 6 — произведение

Чтобы выразить число 3 мы поступили следующим образом:

То есть разделили произведение 6 на множитель 2.

Теперь представим, что в равенстве 3 × 2 = 6 вместо числа 3 располагается переменная x

В этом случае переменная x берет на себя роль неизвестного множимого.

Для нахождения неизвестного множимого предусмотрено следующее правило:

Чтобы найти неизвестное множимое, нужно произведение разделить на множитель.

Что мы и сделали, когда выражали число 3 из равенства 3 × 2 = 6 . Произведение 6 мы разделили на множитель 2.

А сейчас для нахождения неизвестного множимого x , нужно произведение 6 разделить на множитель 2.

Вычисление правой части позволяет нам найти значение переменной x

Это же правило применимо в случае, если переменная x располагается вместо множителя, а не множимого. Представим, что в равенстве 3 × 2 = 6 вместо числа 2 располагается переменная x .

В этом случае переменная x берет на себя роль неизвестного множителя. Для нахождения неизвестного множителя предусмотрено такое же, что и для нахождения неизвестного множимого, а именно деление произведения на известный множитель:

Чтобы найти неизвестный множитель, нужно произведение разделить на множимое.

Что мы и сделали, когда выражали число 2 из равенства 3 × 2 = 6 . Тогда для получения числа 2 мы разделили произведение 6 на множимое 3.

А сейчас для нахождения неизвестного множителя x мы разделили произведение 6 на множимое 3.

Вычисление правой части равенства позволяет узнать чему равно x

Множимое и множитель вместе называют сомножителями. Поскольку правила нахождения множимого и множителя совпадают, мы можем сформулировать общее правило нахождения неизвестного сомножителя:

Чтобы найти неизвестный сомножитель, нужно произведение разделить на известный сомножитель.

Например, решим уравнение 9 × x = 18 . Переменная x является неизвестным сомножителем. Чтобы найти этот неизвестный сомножитель, нужно произведение 18 разделить на известный сомножитель 9

Отсюда .

Решим уравнение x × 3 = 27 . Переменная x является неизвестным сомножителем. Чтобы найти этот неизвестный сомножитель, нужно произведение 27 разделить на известный сомножитель 3

Отсюда .

Вернемся к четвертому примеру из предыдущей темы, где в равенстве требовалось выразить число 15. В этом равенстве число 15 — это делимое, число 5 — делитель, число 3 — частное.

Чтобы выразить число 15 мы поступили следующим образом:

То есть умножили частное 3 на делитель 5.

Теперь представим, что в равенстве вместо числа 15 располагается переменная x

В этом случае переменная x берет на себя роль неизвестного делимого.

Для нахождения неизвестного делимого предусмотрено следующее правило:

Чтобы найти неизвестное делимое, нужно частное умножить на делитель.

Что мы и сделали, когда выражали число 15 из равенства . Чтобы выразить число 15, мы умножили частное 3 на делитель 5.

А сейчас, чтобы найти неизвестное делимое x , нужно частное 3 умножить на делитель 5

Вычислим правую часть получившегося равенства. Так мы узнаем чему равна переменная x .

Теперь представим, что в равенстве вместо числа 5 располагается переменная x .

В этом случае переменная x берет на себя роль неизвестного делителя.

Для нахождения неизвестного делителя предусмотрено следующее правило:

Чтобы найти неизвестный делитель, нужно делимое разделить на частное.

Что мы и сделали, когда выражали число 5 из равенства . Чтобы выразить число 5, мы разделили делимое 15 на частное 3.

А сейчас, чтобы найти неизвестный делитель x , нужно делимое 15 разделить на частное 3

Вычислим правую часть получившегося равенства. Так мы узнаем чему равна переменная x .

Итак, для нахождения неизвестных мы изучили следующие правила:

  • Чтобы найти неизвестное слагаемое, нужно из суммы вычесть известное слагаемое;
  • Чтобы найти неизвестное уменьшаемое, нужно к разности прибавить вычитаемое;
  • Чтобы найти неизвестное вычитаемое, нужно из уменьшаемого вычесть разность;
  • Чтобы найти неизвестное множимое, нужно произведение разделить на множитель;
  • Чтобы найти неизвестный множитель, нужно произведение разделить на множимое;
  • Чтобы найти неизвестное делимое, нужно частное умножить на делитель;
  • Чтобы найти неизвестный делитель, нужно делимое разделить на частное.

Компоненты

Компонентами мы будем называть числа и переменные, входящие в равенство

Так, компонентами сложения являются слагаемые и сумма

Компонентами вычитания являются уменьшаемое, вычитаемое и разность

Компонентами умножения являются множимое, множитель и произведение

Компонентами деления являются делимое, делитель и частное

В зависимости от того, с какими компонентами мы будем иметь дело, будут применяться соответствующие правила нахождения неизвестных. Эти правила мы изучили в предыдущей теме. При решении уравнений желательно знать эти правило наизусть.

Пример 1. Найти корень уравнения 45 + x = 60

45 — слагаемое, x — неизвестное слагаемое, 60 — сумма. Имеем дело с компонентами сложения. Вспоминаем, что для нахождения неизвестного слагаемого, нужно из суммы вычесть известное слагаемое:

Вычислим правую часть, получим значение x равное 15

Значит корень уравнения 45 + x = 60 равен 15.

Чаще всего неизвестное слагаемое необходимо привести к виду при котором его можно было бы выразить.

Пример 2. Решить уравнение

Здесь в отличие от предыдущего примера, неизвестное слагаемое нельзя выразить сразу, поскольку оно содержит коэффициент 2. Наша задача привести это уравнение к виду при котором можно было бы выразить x

В данном примере мы имеем дело с компонентами сложения — слагаемыми и суммой. 2x — это первое слагаемое, 4 — второе слагаемое, 8 — сумма.

При этом слагаемое 2x содержит переменную x . После нахождения значения переменной x слагаемое 2x примет другой вид. Поэтому слагаемое 2x можно полностью принять за неизвестное слагаемое:

Теперь применяем правило нахождения неизвестного слагаемого. Вычитаем из суммы известное слагаемое:

Вычислим правую часть получившегося уравнения:

Мы получили новое уравнение . Теперь мы имеем дело с компонентами умножения: множимым, множителем и произведением. 2 — множимое, x — множитель, 4 — произведение

При этом переменная x является не просто множителем, а неизвестным множителем

Чтобы найти этот неизвестный множитель, нужно произведение разделить на множимое:

Вычислим правую часть, получим значение переменной x

Для проверки найденный корень отправим в исходное уравнение и подставим вместо x

Получили верное числовое равенство. Значит уравнение решено правильно.

Пример 3. Решить уравнение 3x + 9x + 16x = 56

Cразу выразить неизвестное x нельзя. Сначала нужно привести данное уравнение к виду при котором его можно было бы выразить.

Приведем подобные слагаемые в левой части данного уравнения:

Имеем дело с компонентами умножения. 28 — множимое, x — множитель, 56 — произведение. При этом x является неизвестным множителем. Чтобы найти неизвестный множитель, нужно произведение разделить на множимое:

Отсюда x равен 2

Равносильные уравнения

В предыдущем примере при решении уравнения 3x + 9x + 16x = 56 , мы привели подобные слагаемые в левой части уравнения. В результате получили новое уравнение 28x = 56 . Старое уравнение 3x + 9x + 16x = 56 и получившееся новое уравнение 28x = 56 называют равносильными уравнениями, поскольку их корни совпадают.

Уравнения называют равносильными, если их корни совпадают.

Проверим это. Для уравнения 3x + 9x + 16x = 56 мы нашли корень равный 2 . Подставим этот корень сначала в уравнение 3x + 9x + 16x = 56 , а затем в уравнение 28x = 56 , которое получилось в результате приведения подобных слагаемых в левой части предыдущего уравнения. Мы должны получить верные числовые равенства

Согласно порядку действий, в первую очередь выполняется умножение:

Подставим корень 2 во второе уравнение 28x = 56

Видим, что у обоих уравнений корни совпадают. Значит уравнения 3x + 9x + 16x = 56 и 28x = 56 действительно являются равносильными.

Для решения уравнения 3x + 9x + 16x = 56 мы воспользовались одним из тождественных преобразований — приведением подобных слагаемых. Правильное тождественное преобразование уравнения позволило нам получить равносильное уравнение 28x = 56 , которое проще решать.

Из тождественных преобразований на данный момент мы умеем только сокращать дроби, приводить подобные слагаемые, выносить общий множитель за скобки, а также раскрывать скобки. Существуют и другие преобразования, которые следует знать. Но для общего представления о тождественных преобразованиях уравнений, изученных нами тем вполне хватает.

Рассмотрим некоторые преобразования, которые позволяют получить равносильное уравнение

Если к обеим частям уравнения прибавить одно и то же число, то получится уравнение равносильное данному.

Если из обеих частей уравнения вычесть одно и то же число, то получится уравнение равносильное данному.

Другими словами, корень уравнения не изменится, если к обеим частям данного уравнения прибавить (или вычесть из обеих частей) одно и то же число.

Пример 1. Решить уравнение

Вычтем из обеих частей уравнения число 10

Приведем подобные слагаемые в обеих частях:

Получили уравнение 5x = 10 . Имеем дело с компонентами умножения. Чтобы найти неизвестный сомножитель x , нужно произведение 10 разделить на известный сомножитель 5.

Отсюда .

Вернемся к исходному уравнению и подставим вместо x найденное значение 2

Получили верное числовое равенство. Значит уравнение решено правильно.

Решая уравнение мы вычли из обеих частей уравнения число 10 . В результате получили равносильное уравнение . Корень этого уравнения, как и уравнения так же равен 2

Пример 2. Решить уравнение 4(x + 3) = 16

Раскроем скобки в левой части равенства:

Вычтем из обеих частей уравнения число 12

Приведем подобные слагаемые в обеих частях уравнения:

В левой части останется 4x , а в правой части число 4

Получили уравнение 4x = 4 . Имеем дело с компонентами умножения. Чтобы найти неизвестный сомножитель x , нужно произведение 4 разделить на известный сомножитель 4

Отсюда

Вернемся к исходному уравнению 4(x + 3) = 16 и подставим вместо x найденное значение 1

Получили верное числовое равенство. Значит уравнение решено правильно.

Решая уравнение 4(x + 3) = 16 мы вычли из обеих частей уравнения число 12 . В результате получили равносильное уравнение 4x = 4 . Корень этого уравнения, как и уравнения 4(x + 3) = 16 так же равен 1

Пример 3. Решить уравнение

Раскроем скобки в левой части равенства:

Прибавим к обеим частям уравнения число 8

Приведем подобные слагаемые в обеих частях уравнения:

В левой части останется 2x , а в правой части число 9

В получившемся уравнении 2x = 9 выразим неизвестное слагаемое x

Отсюда

Вернемся к исходному уравнению и подставим вместо x найденное значение 4,5

Получили верное числовое равенство. Значит уравнение решено правильно.

Решая уравнение мы прибавили к обеим частям уравнения число 8. В результате получили равносильное уравнение . Корень этого уравнения, как и уравнения так же равен 4,5

Следующее правило, которое позволяет получить равносильное уравнение, выглядит следующим образом

Если в уравнении перенести слагаемое из одной части в другую, изменив его знак, то получится уравнение равносильное данному.

То есть корень уравнения не изменится, если мы перенесем слагаемое из одной части уравнения в другую, изменив его знак. Это свойство является одним из важных и одним из часто используемых при решении уравнений.

Рассмотрим следующее уравнение:

Корень данного уравнения равен 2. Подставим вместо x этот корень и проверим получается ли верное числовое равенство

Получается верное равенство. Значит число 2 действительно является корнем уравнения .

Теперь попробуем поэкспериментировать со слагаемыми этого уравнения, перенося их из одной части в другую, изменяя знаки.

Например, слагаемое 3x располагается в левой части равенства. Перенесём его в правую часть, изменив знак на противоположный:

Получилось уравнение 12 = 9x − 3x . Приведем подобные слагаемые в правой части данного уравнения:

Имеем дело с компонентами умножения. Переменная x является неизвестным сомножителем. Найдём этот известный сомножитель:

Отсюда x = 2 . Как видим, корень уравнения не изменился. Значит уравнения 12 + 3x = 9x и 12 = 9x − 3x являются равносильными.

На самом деле данное преобразование является упрощенным методом предыдущего преобразования, где к обеим частям уравнения прибавлялось (или вычиталось) одно и то же число.

Мы сказали, что в уравнении 12 + 3x = 9x слагаемое 3x было перенесено в правую часть, изменив знак. В реальности же происходило следующее: из обеих частей уравнения вычли слагаемое 3x

Затем в левой части были приведены подобные слагаемые и получено уравнение 12 = 9x − 3x. Затем опять были приведены подобные слагаемые, но уже в правой части, и получено уравнение 12 = 6x.

Но так называемый «перенос» более удобен для подобных уравнений, поэтому он и получил такое широкое распространение. Решая уравнения, мы часто будем пользоваться именно этим преобразованием.

Равносильными также являются уравнения 12 + 3x = 9x и 3x − 9x = −12 . В этот раз в уравнении 12 + 3x = 9x слагаемое 12 было перенесено в правую часть, а слагаемое 9x в левую. Не следует забывать, что знаки этих слагаемых были изменены во время переноса

Следующее правило, которое позволяет получить равносильное уравнение, выглядит следующим образом:

Если обе части уравнения умножить или разделить на одно и то же число, не равное нулю, то получится уравнение равносильное данному.

Другими словами, корни уравнения не изменятся, если обе его части умножить или разделить на одно и то же число. Это действие часто применяется тогда, когда нужно решить уравнение содержащее дробные выражения.

Сначала рассмотрим примеры, в которых обе части уравнения будут умножаться на одно и то же число.

Пример 1. Решить уравнение

При решении уравнений, содержащих дробные выражения, сначала принято упростить это уравнение.

В данном случае мы имеем дело именно с таким уравнением. В целях упрощения данного уравнения обе его части можно умножить на 8:

Мы помним, что для умножения дроби на число, нужно числитель данной дроби умножить на это число. У нас имеются две дроби и каждая из них умножается на число 8. Наша задача умножить числители дробей на это число 8

Теперь происходит самое интересное. В числителях и знаменателях обеих дробей содержится множитель 8, который можно сократить на 8. Это позволит нам избавиться от дробного выражения:

В результате останется простейшее уравнение

Ну и нетрудно догадаться, что корень этого уравнения равен 4

Вернемся к исходному уравнению и подставим вместо x найденное значение 4

Получается верное числовое равенство. Значит уравнение решено правильно.

При решении данного уравнения мы умножили обе его части на 8. В результате получили уравнение . Корень этого уравнения, как и уравнения равен 4. Значит эти уравнения равносильны.

Множитель на который умножаются обе части уравнения принято записывать перед частью уравнения, а не после неё. Так, решая уравнение , мы умножили обе части на множитель 8 и получили следующую запись:

От этого корень уравнения не изменился, но если бы мы сделали это находясь в школе, то нам сделали бы замечание, поскольку в алгебре множитель принято записывать перед тем выражением, с которым он перемножается. Поэтому умножение обеих частей уравнения на множитель 8 желательно переписать следующим образом:

Пример 2. Решить уравнение

Умнóжим обе части уравнения на 15

В левой части множители 15 можно сократить на 15, а в правой части множители 15 и 5 можно сократить на 5

Перепишем то, что у нас осталось:

Раскроем скобки в правой части уравнения:

Перенесем слагаемое x из левой части уравнения в правую часть, изменив знак. А слагаемое 15 из правой части уравнения перенесем в левую часть, опять же изменив знак:

Приведем подобные слагаемые в обеих частях, получим

Имеем дело с компонентами умножения. Переменная x является неизвестным сомножителем. Найдём этот известный сомножитель:

Отсюда

Вернемся к исходному уравнению и подставим вместо x найденное значение 5

Получается верное числовое равенство. Значит уравнение решено правильно. При решении данного уравнения мы умножили обе го части на 15 . Далее выполняя тождественные преобразования, мы получили уравнение 10 = 2x . Корень этого уравнения, как и уравнения равен 5 . Значит эти уравнения равносильны.

Пример 3. Решить уравнение

Умнóжим обе части уравнения на 3

В левой части можно сократить две тройки, а правая часть будет равна 18

Останется простейшее уравнение . Имеем дело с компонентами умножения. Переменная x является неизвестным сомножителем. Найдём этот известный сомножитель:

Отсюда

Вернемся к исходному уравнению и подставим вместо x найденное значение 9

Получается верное числовое равенство. Значит уравнение решено правильно.

Пример 4. Решить уравнение

Умнóжим обе части уравнения на 6

В левой части уравнения раскроем скобки. В правой части множитель 6 можно поднять в числитель:

Сократим в обеих частях уравнениях то, что можно сократить:

Перепишем то, что у нас осталось:

Раскроем скобки в обеих частях уравнения:

Воспользуемся переносом слагаемых. Слагаемые, содержащие неизвестное x , сгруппируем в левой части уравнения, а слагаемые свободные от неизвестных — в правой:

Приведем подобные слагаемые в обеих частях:

Теперь найдем значение переменной x . Для этого разделим произведение 28 на известный сомножитель 7

Вернемся к исходному уравнению и подставим вместо x найденное значение 4

Получилось верное числовое равенство. Значит уравнение решено правильно.

Пример 5. Решить уравнение

Раскроем скобки в обеих частях уравнения там, где это можно:

Умнóжим обе части уравнения на 15

Раскроем скобки в обеих частях уравнения:

Сократим в обеих частях уравнения, то что можно сократить:

Перепишем то, что у нас осталось:

Раскроем скобки там, где это можно:

Воспользуемся переносом слагаемых. Слагаемые, содержащие неизвестное, сгруппируем в левой части уравнения, а слагаемые, свободные от неизвестных — в правой. Не забываем, что во время переноса, слагаемые меняют свои знаки на противоположные:

Приведем подобные слагаемые в обеих частях уравнения:

Найдём значение x

В получившемся ответе можно выделить целую часть:

Вернемся к исходному уравнению и подставим вместо x найденное значение

Получается довольно громоздкое выражение. Воспользуемся переменными. Левую часть равенства занесем в переменную A , а правую часть равенства в переменную B

Наша задача состоит в том, чтобы убедиться равна ли левая часть правой. Другими словами, доказать равенство A = B

Найдем значение выражения, находящегося в переменной А.

Значение переменной А равно . Теперь найдем значение переменной B . То есть значение правой части нашего равенства. Если и оно равно , то уравнение будет решено верно

Видим, что значение переменной B , как и значение переменной A равно . Это значит, что левая часть равна правой части. Отсюда делаем вывод, что уравнение решено правильно.

Теперь попробуем не умножать обе части уравнения на одно и то же число, а делить.

Рассмотрим уравнение 30x + 14x + 14 = 70x − 40x + 42 . Решим его обычным методом: слагаемые, содержащие неизвестные, сгруппируем в левой части уравнения, а слагаемые, свободные от неизвестных — в правой. Далее выполняя известные тождественные преобразования, найдем значение x

Подставим найденное значение 2 вместо x в исходное уравнение:

Теперь попробуем разделить все слагаемые уравнения 30x + 14x + 14 = 70x − 40x + 42 на какое-нибудь число. Замечаем, что все слагаемые этого уравнения имеют общий множитель 2. На него и разделим каждое слагаемое:

Выполним сокращение в каждом слагаемом:

Перепишем то, что у нас осталось:

Решим это уравнение, пользуясь известными тождественными преобразованиями:

Получили корень 2 . Значит уравнения 15x + 7x + 7 = 35x − 20x + 21 и 30x + 14x + 14 = 70x − 40x + 42 равносильны.

Деление обеих частей уравнения на одно и то же число позволяет освобождать неизвестное от коэффициента. В предыдущем примере когда мы получили уравнение 7x = 14 , нам потребовалось разделить произведение 14 на известный сомножитель 7. Но если бы мы в левой части освободили неизвестное от коэффициента 7, корень нашелся бы сразу. Для этого достаточно было разделить обе части на 7

Этим методом мы тоже будем пользоваться часто.

Умножение на минус единицу

Если обе части уравнения умножить на минус единицу, то получится уравнение равносильное данному.

Это правило следует из того, что от умножения (или деления) обеих частей уравнения на одно и то же число, корень данного уравнения не меняется. А значит корень не поменяется если обе его части умножить на −1 .

Данное правило позволяет поменять знаки всех компонентов, входящих в уравнение. Для чего это нужно? Опять же, чтобы получить равносильное уравнение, которое проще решать.

Рассмотрим уравнение . Чему равен корень этого уравнения?

Прибавим к обеим частям уравнения число 5

Приведем подобные слагаемые:

А теперь вспомним про коэффициент буквенного выражения. Что же представляет собой левая часть уравнения . Это есть произведение минус единицы и переменной x

То есть минус, стоящий перед переменной x, относится не к самой переменной x , а к единице, которую мы не видим, поскольку коэффициент 1 принято не записывать. Это означает, что уравнение на самом деле выглядит следующим образом:

Имеем дело с компонентами умножения. Чтобы найти х , нужно произведение −5 разделить на известный сомножитель −1 .

или разделить обе части уравнения на −1 , что еще проще

Итак, корень уравнения равен 5 . Для проверки подставим его в исходное уравнение. Не забываем, что в исходном уравнении минус стоящий перед переменной x относится к невидимой единице

Получилось верное числовое равенство. Значит уравнение решено верно.

Теперь попробуем умножить обе части уравнения на минус единицу:

После раскрытия скобок в левой части образуется выражение , а правая часть будет равна 10

Корень этого уравнения, как и уравнения равен 5

Значит уравнения и равносильны.

Пример 2. Решить уравнение

В данном уравнении все компоненты являются отрицательными. С положительными компонентами работать удобнее, чем с отрицательными, поэтому поменяем знаки всех компонентов, входящих в уравнение . Для этого умнóжим обе части данного уравнения на −1 .

Понятно, что от умножения на −1 любое число поменяет свой знак на противоположный. Поэтому саму процедуру умножения на −1 и раскрытие скобок подробно не расписывают, а сразу записывают компоненты уравнения с противоположными знаками.

Так, умножение уравнения на −1 можно записать подробно следующим образом:

либо можно просто поменять знаки всех компонентов:

Получится то же самое, но разница будет в том, что мы сэкономим себе время.

Итак, умножив обе части уравнения на −1 , мы получили уравнение . Решим данное уравнение. Из обеих частей вычтем число 4 и разделим обе части на 3

Когда корень найден, переменную обычно записывают в левой части, а её значение в правой, что мы и сделали.

Пример 3. Решить уравнение

Умнóжим обе части уравнения на −1 . Тогда все компоненты поменяют свои знаки на противоположные:

Из обеих частей получившегося уравнения вычтем 2x и приведем подобные слагаемые:

Прибавим к обеим частям уравнения единицу и приведем подобные слагаемые:

Приравнивание к нулю

Недавно мы узнали, что если в уравнении перенести слагаемое из одной части в другую, изменив его знак, то получится уравнение равносильное данному.

А что будет если перенести из одной части в другую не одно слагаемое, а все слагаемые? Верно, в той части откуда забрали все слагаемые останется ноль. Иными словами, не останется ничего.

В качестве примера рассмотрим уравнение . Решим данное уравнение, как обычно — слагаемые, содержащие неизвестные сгруппируем в одной части, а числовые слагаемые, свободные от неизвестных оставим в другой. Далее выполняя известные тождественные преобразования, найдем значение переменной x

Теперь попробуем решить это же уравнение, приравняв все его компоненты к нулю. Для этого перенесем все слагаемые из правой части в левую, изменив знаки:

Приведем подобные слагаемые в левой части:

Прибавим к обеим частям 77 , и разделим обе части на 7

Альтернатива правилам нахождения неизвестных

Очевидно, что зная о тождественных преобразованиях уравнений, можно не заучивать наизусть правила нахождения неизвестных.

К примеру, для нахождения неизвестного в уравнении мы произведение 10 делили на известный сомножитель 2

Но если в уравнении обе части разделить на 2 корень найдется сразу. В левой части уравнения в числителе множитель 2 и в знаменателе множитель 2 сократятся на 2. А правая часть будет равна 5

Уравнения вида мы решали выражая неизвестное слагаемое:

Но можно воспользоваться тождественными преобразованиями, которые мы сегодня изучили. В уравнении слагаемое 4 можно перенести в правую часть, изменив знак:

Далее разделить обе части на 2

В левой части уравнения сократятся две двойки. Правая часть будет равна 2. Отсюда .

Либо можно было из обеих частей уравнения вычесть 4. Тогда получилось бы следующее:

В случае с уравнениями вида удобнее делить произведение на известный сомножитель. Сравним оба решения:

Первое решение намного короче и аккуратнее. Второе решение можно значительно укоротить, если выполнить деление в уме.

Тем не менее, необходимо знать оба метода, и только затем использовать тот, который больше нравится.

Когда корней несколько

Уравнение может иметь несколько корней. Например уравнение x(x + 9) = 0 имеет два корня: 0 и −9 .

В уравнении x(x + 9) = 0 нужно было найти такое значение x при котором левая часть была бы равна нулю. В левой части этого уравнения содержатся выражения x и (x + 9) , которые являются сомножителями. Из законов умножения мы знаем, что произведение равно нулю, если хотя бы один из сомножителей равен нулю (или первый сомножитель или второй).

То есть в уравнении x(x + 9) = 0 равенство будет достигаться, если x будет равен нулю или (x + 9) будет равно нулю.

Приравняв к нулю оба этих выражения, мы сможем найти корни уравнения x(x + 9) = 0 . Первый корень, как видно из примера, нашелся сразу. Для нахождения второго корня нужно решить элементарное уравнение x + 9 = 0 . Несложно догадаться, что корень этого уравнения равен −9 . Проверка показывает, что корень верный:

Пример 2. Решить уравнение

Данное уравнение имеет два корня: 1 и 2. Левая часть уравнения является произведение выражений (x − 1) и (x − 2) . А произведение равно нулю, если хотя бы один из сомножителей равен нулю (или сомножитель (x − 1) или сомножитель (x − 2) ).

Найдем такое x при котором выражения (x − 1) или (x − 2) обращаются в нули:

Подставляем по-очереди найденные значения в исходное уравнение и убеждаемся, что при этих значениях левая часть равняется нулю:

Когда корней бесконечно много

Уравнение может иметь бесконечно много корней. То есть подставив в такое уравнение любое число, мы получим верное числовое равенство.

Пример 1. Решить уравнение

Корнем данного уравнения является любое число. Если раскрыть скобки в левой части уравнения и привести подобные слагаемые, то получится равенство 14 = 14 . Это равенство будет получаться при любом x

Пример 2. Решить уравнение

Корнем данного уравнения является любое число. Если раскрыть скобки в левой части уравнения, то получится равенство 10x + 12 = 10x + 12. Это равенство будет получаться при любом x

Когда корней нет

Случается и так, что уравнение вовсе не имеет решений, то есть не имеет корней. Например уравнение не имеет корней, поскольку при любом значении x , левая часть уравнения не будет равна правой части. Например, пусть . Тогда уравнение примет следующий вид

Пусть

Пример 2. Решить уравнение

Раскроем скобки в левой части равенства:

Приведем подобные слагаемые:

Видим, что левая часть не равна правой части. И так будет при любом значении y . Например, пусть y = 3 .

Буквенные уравнения

Уравнение может содержать не только числа с переменными, но и буквы.

Например, формула нахождения скорости является буквенным уравнением:

Данное уравнение описывает скорость движения тела при равноускоренном движении.

Полезным навыком является умение выразить любой компонент, входящий в буквенное уравнение. Например, чтобы из уравнения определить расстояние, нужно выразить переменную s .

Умнóжим обе части уравнения на t

В правой части переменные t сократим на t и перепишем то, что у нас осталось:

В получившемся уравнении левую и правую часть поменяем местами:

У нас получилась формула нахождения расстояния, которую мы изучали ранее.

Попробуем из уравнения определить время. Для этого нужно выразить переменную t .

Умнóжим обе части уравнения на t

В правой части переменные t сократим на t и перепишем то, что у нас осталось:

В получившемся уравнении v × t = s обе части разделим на v

В левой части переменные v сократим на v и перепишем то, что у нас осталось:

У нас получилась формула определения времени, которую мы изучали ранее.

Предположим, что скорость поезда равна 50 км/ч

А расстояние равно 100 км

Тогда буквенное уравнение примет следующий вид

Из этого уравнения можно найти время. Для этого нужно суметь выразить переменную t . Можно воспользоваться правилом нахождения неизвестного делителя, разделив делимое на частное и таким образом определить значение переменной t

либо можно воспользоваться тождественными преобразованиями. Сначала умножить обе части уравнения на t

Затем разделить обе части на 50

Пример 2. Дано буквенное уравнение . Выразите из данного уравнения x

Вычтем из обеих частей уравнения a

Разделим обе части уравнения на b

Теперь, если нам попадется уравнение вида a + bx = c , то у нас будет готовое решение. Достаточно будет подставить в него нужные значения. Те значения, которые будут подставляться вместо букв a, b, c принято называть параметрами. А уравнения вида a + bx = c называют уравнением с параметрами. В зависимости от параметров, корень будет меняться.

Решим уравнение 2 + 4x = 10 . Оно похоже на буквенное уравнение a + bx = c . Вместо того, чтобы выполнять тождественные преобразования, мы можем воспользоваться готовым решением. Сравним оба решения:

Видим, что второе решение намного проще и короче.

Для готового решения необходимо сделать небольшое замечание. Параметр b не должен быть равным нулю (b ≠ 0) , поскольку деление на ноль на допускается.

Пример 3. Дано буквенное уравнение . Выразите из данного уравнения x

Раскроем скобки в обеих частях уравнения

Воспользуемся переносом слагаемых. Параметры, содержащие переменную x , сгруппируем в левой части уравнения, а параметры свободные от этой переменной — в правой.

В левой части вынесем за скобки множитель x

Разделим обе части на выражение a − b

В левой части числитель и знаменатель можно сократить на a − b . Так окончательно выразится переменная x

Теперь, если нам попадется уравнение вида a(x − c) = b(x + d) , то у нас будет готовое решение. Достаточно будет подставить в него нужные значения.

Допустим нам дано уравнение 4(x − 3) = 2(x + 4) . Оно похоже на уравнение a(x − c) = b(x + d) . Решим его двумя способами: при помощи тождественных преобразований и при помощи готового решения:

Для удобства вытащим из уравнения 4(x − 3) = 2(x + 4) значения параметров a, b, c, d . Это позволит нам не ошибиться при подстановке:

Как и в прошлом примере знаменатель здесь не должен быть равным нулю (a − b ≠ 0) . Если нам встретится уравнение вида a(x − c) = b(x + d) в котором параметры a и b будут одинаковыми, мы сможем не решая его сказать, что у данного уравнения корней нет, поскольку разность одинаковых чисел равна нулю.

Например, уравнение 2(x − 3) = 2(x + 4) является уравнением вида a(x − c) = b(x + d) . В уравнении 2(x − 3) = 2(x + 4) параметры a и b одинаковые. Если мы начнём его решать, то придем к тому, что левая часть не будет равна правой части:

Пример 4. Дано буквенное уравнение . Выразите из данного уравнения x

Приведем левую часть уравнения к общему знаменателю:

Умнóжим обе части на a

В левой части x вынесем за скобки

Разделим обе части на выражение (1 − a)

Линейные уравнения с одним неизвестным

Рассмотренные в данном уроке уравнения называют линейными уравнениями первой степени с одним неизвестным.

Если уравнение дано в первой степени, не содержит деления на неизвестное, а также не содержит корней из неизвестного, то его можно назвать линейным. Мы еще не изучали степени и корни, поэтому чтобы не усложнять себе жизнь, слово «линейный» будем понимать как «простой».

Большинство уравнений, решенных в данном уроке, в конечном итоге сводились к простейшему уравнению, в котором нужно было произведение разделить на известный сомножитель. Таковым к примеру является уравнение 2 (x + 3) = 16 . Давайте решим его.

Раскроем скобки в левой части уравнения, получим 2 x + 6 = 16. Перенесем слагаемое 6 в правую часть, изменив знак. Тогда получим 2 x = 16 − 6. Вычислим правую часть, получим 2x = 10. Чтобы найти x , разделим произведение 10 на известный сомножитель 2. Отсюда x = 5.

Уравнение 2 (x + 3) = 16 является линейным. Оно свелось к уравнению 2x = 10 , для нахождения корня которого потребовалось разделить произведение на известный сомножитель. Такое простейшее уравнение называют линейным уравнением первой степени с одним неизвестным в каноническом виде. Слово «канонический» является синонимом слов «простейший» или «нормальный».

Линейное уравнение первой степени с одним неизвестным в каноническом виде называют уравнение вида ax = b.

Полученное нами уравнение 2x = 10 является линейным уравнением первой степени с одним неизвестным в каноническом виде. У этого уравнения первая степень, одно неизвестное, оно не содержит деления на неизвестное и не содержит корней из неизвестного, и представлено оно в каноническом виде, то есть в простейшем виде при котором легко можно определить значение x . Вместо параметров a и b в нашем уравнении содержатся числа 2 и 10. Но подобное уравнение может содержать и другие числа: положительные, отрицательные или равные нулю.

Если в линейном уравнении a = 0 и b = 0 , то уравнение имеет бесконечно много корней. Действительно, если a равно нулю и b равно нулю, то линейное уравнение ax = b примет вид 0x = 0 . При любом значении x левая часть будет равна правой части.

Если в линейном уравнении a = 0 и b ≠ 0 , то уравнение корней не имеет. Действительно, если a равно нулю и b равно какому-нибудь числу, не равному нулю, скажем числу 5, то уравнение ax = b примет вид 0x = 5 . Левая часть будет равна нулю, а правая часть пяти. А ноль не равен пяти.

Если в линейном уравнении a ≠ 0 , и b равно любому числу, то уравнение имеет один корень. Он определяется делением параметра b на параметр a

Действительно, если a равно какому-нибудь числу, не равному нулю, скажем числу 3 , и b равно какому-нибудь числу, скажем числу 6 , то уравнение примет вид .
Отсюда .

Существует и другая форма записи линейного уравнения первой степени с одним неизвестным. Выглядит она следующим образом: ax − b = 0 . Это то же самое уравнение, что и ax = b , но параметр b перенесен в левую часть с противоположным знаком. Такие уравнение мы тоже решали в данном уроке. Например, уравнение 7x − 77 = 0 . Уравнение вида ax − b = 0 называют линейным уравнением первой степени с одним неизвестным в общем виде.

В будущем после изучения рациональных выражений, мы рассмотрим такие понятия, как посторонние корни и потеря корней. А пока рассмотренного в данном уроке будет достаточным.

источники:

http://skysmart.ru/articles/mathematic/reshenie-prostyh-linejnyh-uravnenij

http://spacemath.xyz/obshhie-svedeniya-ob-uravneniyah/

«Умственную самодеятельность, сообразительность и смекалку нельзя ни
«вдолбить», ни «вложить» ни в чью голову. Результаты надёжны лишь тогда, когда
введение в область математических знаний осуществляется в лёгкой и приятной
форме, на предметах и примерах обыденной и повседневной обстановки, подобранных
с надлежащим остроумием и занимательностью».

Е.И. Игнатьев «В царстве смекалки»

Цель обучающая: Научить анализировать условие задачи, выбирать
рациональные способы решения, составлять уравнение, решать его, проверять
правильность решения.

Цель воспитательная: Развитие абстрактного и логического мышления.

Ход урока

I. Актуализация опорных знаний.

1) Что называется уравнением? Что называется корнем уравнения? Что значит
решить уравнение?

2) Решите уравнение
.

Решение: а)
;
; x=
;

б) НОЗ=6;
;
;
;
;

3) Составьте буквенное выражение.

Наташа купила блокнот за m рублей, книгу на 20 рублей дороже блокнота и ручку
в 2.5 раза дешевле книги. Сколько рублей стоит ручка?

Решение: (m+20) рублей цена книги,
рублей
цена ручки.

Решить задачу.

4) Из двух сёл, расстояние между которыми 10 км навстречу друг другу вышли
мальчик и девочка и встретились через 2 часа. Скорость мальчика 3 км/час.
Найдите скорость девочки.

Решение: а) 3*2=6(км) прошел мальчик, 10-6=4(км) прошла девочка,
4:2=2(км/ч) скорость девочки

б) 10:2=5(км/ч) скорость сближения, 5-3=2(км/ч) скорость девочки.

II. Сообщение темы и цели урока.

III. Решение задач составлением уравнения.

1) Андрей старше Олега на 4 года, а Олег старше Бориса в 1,5 раза. Вместе им
36 лет. Сколько лет каждому из них?

Первый ряд решает задачу, взяв за неизвестную величину возраст Андрея, второй
ряд — возраст Олега, третий ряд-возраст Бориса. А затем каждый ряд объясняет
своё решение у доски.

Условие задачи Решение уравнения Проверка
Х лет Андрею,

(х-4) лет Олегу,


лет Борису,

16+12+8=36
Х лет Олегу

Х+4 тут Андрею


лет Борису

12+16+8=36
Х лет Борису

1,5х лет Олегу

(1,5х+4) лет Андрею

8+12+16=36

Вывод.

Если в задаче несколько неизвестных величин, лучше обозначить буквой
наименьшую из них.

2) Два пешехода вышли одновременно навстречу друг другу из двух посёлков и
встретились через 3ч. Расстояние между посёлками 30 км. Найдите скорость каждого
пешехода, если у одного она на 2 км/ч меньше, чем у другого.

а) Ученики решают задачу составлением уравнения.

Х км/ч – скорость 1-го пешехода

(х+2) км/ч – скорость 2-го пешехода

3х км прошел 1-й пешеход

3(х+2) км прошел 2-й пешеход

3х+3(х+2)=30 4
3х+3х+6=30 +6
6х+6=30 +12
6х=24 +18
Х=4 =30

б) Затем решают арифметическим способом.

30:3=10(км/ч)-скорость сближения.

10-2=8(км/ч)-две скорости 1-го пешехода.

8:2=4(км/ч)-скорость 1-го пешехода.

4+2=6(км/ч)-скорость 2-го пешехода.

Вывод. Арифметическое решение задачи более рациональное

3)Для распечатки 340 страниц были использованы две копировальные машины.
Первая машина работала 10 минут, а вторая 15 минут. Сколько страниц в минуту
печатает каждая машина, если первая печатает на 4 страницы больше, чем вторая?

Снова дети решают задачу алгебраически и арифметически.

А) х страниц за 1 минуту печатает 2-я машина,

(х+4) страниц за 1 минуту напечатает 1-я машина,

15х страниц напечатает 2 машина,

10(х+4) страниц напечатает 2 машина,

15х+10(х+4=340 12
15х+10х+40=340 +16
25х+40=340 +180
25х=300 +160
Х=12 =340

б) 10*4=40(стр) на столько страниц больше напечатает 1-я машина за 10 минут

340-40=300(стр) напечатали бы обе машины вместе, если бы у них были
одинаковые скорости.

10+15=25(мин) работали обе машины.

300:25=12(стр) за 1-у минуту печатает 2-я машина.

12+4=16 (стр) за 1-у минуту печатает 1-я машина.

IV. Итоги урока.

При решении задач с помощью уравнения поступают следующим образом:

  1. Внимательно читают условие задачи.
  2. Обозначают неизвестную величину буквой.
  3. Переводят условие задачи на алгебраический язык.
  4. Составляют уравнение.
  5. Решают уравнение.
  6. Проверяют правильность решения

V. Домашнее задание.

Решить задачу алгебраически и арифметически.

Охотничья собака спугнула зайца, который сидел под кустом в 150м от неё.
Через сколько минут собака догонит зайца, если она пробегает за 6 минут 3,6км, а
заяц только 3км?

Литература:

  1. Л.В. Кузнецова, Е.А. Бунимович, Б.П. Пигарев, С.Б. Суворова «Алгебра»
    сборник заданий для проведения письменного экзамена по алгебре за курс основной
    школы. Москва. «Дрофа». 2001г; стр 43, работа №31, вариант 1(5); стр 50, работа №38, вариант 2(7); стр 65, работа №48, вариант 1(6).
  2. Составители Т.А. Братусь, Н.А. Жарковская, Е.А Рисс, Т.Е Савелова «Детский
    математический календарь 2001-2002». Санкт — Питербург стр10.
  3. Н.Т. Кострикина «Задачи повышенной трудности в курсе алгебры 7-9 классов».
    Москва. «Просвещение» 1991 стр5-19.

Понравилась статья? Поделить с друзьями:
  • Нет доступа к закрытому ключу как исправить
  • Как найти двигатель на мотоблок
  • Как найти скрытые приложения на андроиде самсунг
  • Как найти свой заказ на яндекс еде
  • Как найти сторону описанного треугольника через радиус